linalg2020/docs/loesungen.tex

4154 lines
160 KiB
TeX
Raw Blame History

This file contains ambiguous Unicode characters

This file contains Unicode characters that might be confused with other characters. If you think that this is intentional, you can safely ignore this warning. Use the Escape button to reveal them.

%% ********************************************************************************
%% AUTHOR: Raj Dahya
%% CREATED: November 2020
%% EDITED: —
%% TYPE: Notizen
%% TITLE: Lösungen zu diversen Aufgaben im Kurs
%% DOI: —
%% DEPARTMENT: Fakultät for Mathematik und Informatik
%% INSTITUTE: Universität Leipzig
%% ********************************************************************************
%% ********************************************************************************
%% DOCUMENT STRUCTURE:
%% ~~~~~~~~~~~~~~~~~~~
%%
%% — root.tex;
%% |
%% — parameters.tex;
%% |
%% — src/index.tex;
%% |
%% — ########;
%% |
%% — src/setup-type.tex;
%% |
%% — src/setup-packages.tex;
%% |
%% — src/setup-parameters.tex;
%% |
%% — src/setup-macros.tex;
%% |
%% — src/setup-environments.tex;
%% |
%% — src/setup-layout.tex;
%% |
%% — src/setup-localmacros.tex;
%% |
%% — front/index.tex;
%% |
%% — front/title.tex;
%% |
%% — front/foreword.tex;
%% |
%% — front/contents.tex;
%% |
%% — body/index.tex;
%% |
%% — body/uebung/ueb1.tex;
%% |
%% — body/uebung/ueb2.tex;
%% |
%% — body/uebung/ueb3.tex;
%% |
%% — body/ska/ska4.tex;
%% |
%% — body/quizzes/quiz1.tex;
%% |
%% — body/quizzes/quiz2.tex;
%% |
%% — body/quizzes/quiz3.tex;
%% |
%% — back/index.tex;
%% |
%% — ./back/quelle.bib;
%%
%% DOCUMENT-RANDOM-SEED: 5637845
%% ********************************************************************************
%% ********************************************************************************
%% FILE: root.tex
%% ********************************************************************************
%% ********************************************************************************
%% FILE: parameters.tex
%% ********************************************************************************
%% ********************************************************************************
%% FILE: src/index.tex
%% ********************************************************************************
%% ********************************************************************************
%% FILE: ########
%% ********************************************************************************
\makeatletter
%% ********************************************************************************
%% FILE: src/setup-type.tex
%% ********************************************************************************
\documentclass[
12pt,
a4paper,
oneside,
openright,
center,
chapterbib,
crosshair,
fleqn,
headcount,
headline,
indent,
indentfirst=false,
portrait,
phonetic,
oldernstyle,
onecolumn,
sfbold,
upper,
]{scrbook}
%% ********************************************************************************
%% FILE: src/setup-packages.tex
%% ********************************************************************************
\PassOptionsToPackage{T2A,OT1}{fontenc} % T1,OT1,T2A,OT2
\PassOptionsToPackage{utf8}{inputenc} % utf8
\PassOptionsToPackage{british,english,ngerman,russian}{babel}
\PassOptionsToPackage{
english,
ngerman,
russian,
capitalise,
}{cleveref}
\PassOptionsToPackage{
bookmarks=true,
bookmarksopen=false,
bookmarksopenlevel=0,
bookmarkstype=toc,
colorlinks=false,
raiselinks=true,
hyperfigures=true,
}{hyperref}
\PassOptionsToPackage{
reset,
left=1in,
right=1in,
top=20mm,
bottom=20mm,
heightrounded,
}{geometry}
\PassOptionsToPackage{
framemethod=TikZ,
}{mdframed}
\PassOptionsToPackage{normalem}{ulem}
\PassOptionsToPackage{
amsmath,
thmmarks,
}{ntheorem}
\PassOptionsToPackage{table}{xcolor}
\PassOptionsToPackage{
all,
color,
curve,
frame,
import,
knot,
line,
movie,
rotate,
textures,
tile,
tips,
web,
xdvi,
}{xy}
\usepackage{amsfonts}
\usepackage{amsmath}
\usepackage{amssymb}
\usepackage{ntheorem} % <— muss nach den ams* Packages vorkommen!!
\usepackage{array}
\usepackage{babel}
\usepackage{bbding}
\usepackage{bbm}
\usepackage{calc}
\usepackage{sectsty}
\usepackage{titlesec}
\usepackage{fancyhdr}
\usepackage{footmisc}
\usepackage{geometry}
\usepackage{graphicx}
\usepackage{ifpdf}
\usepackage{ifthen}
\usepackage{ifnextok}
\usepackage{longtable}
\usepackage{nameref}
\usepackage{nowtoaux}
\usepackage{paralist}
\usepackage{enumerate} %% nach [paralist]
\usepackage{pgf}
\usepackage{pgfplots}
\usepackage{proof}
\usepackage{refcount}
\usepackage{relsize}
\usepackage{savesym}
\usepackage{stmaryrd}
\usepackage{yfonts} %% <— Altgotische Fonts
\usepackage{tikz}
\usepackage{xy}
\usepackage{undertilde}
\usepackage{ulem} %% < f\"ur besseren \underline-Befehl (\ul)
\usepackage{xcolor}
\usepackage{xspace}
\usepackage{xstring}
\usepackage{hyperref}
\usepackage{cleveref} % must vor hyperref geladen werden.
\pgfplotsset{compat=newest}
\usetikzlibrary{
angles,
arrows,
automata,
calc,
decorations,
decorations.pathmorphing,
decorations.pathreplacing,
math,
positioning,
patterns,
quotes,
snakes,
}
%% \var ≈ alter Befehl
%% \xvar ≈ wie das neue Package \var interpretieren soll.
\savesymbol{Diamond}
\savesymbol{emptyset}
\savesymbol{ggg}
\savesymbol{int}
\savesymbol{lll}
\savesymbol{RectangleBold}
\savesymbol{langle}
\savesymbol{rangle}
\savesymbol{hookrightarrow}
\savesymbol{hookleftarrow}
\savesymbol{Asterisk}
\usepackage{mathabx}
\usepackage{wasysym}
\let\varemptyset=\emptyset
\restoresymbol{x}{Diamond}
\restoresymbol{x}{emptyset}
\restoresymbol{x}{ggg}
\restoresymbol{x}{int}
\restoresymbol{x}{lll}
\restoresymbol{x}{RectangleBold}
\restoresymbol{x}{langle}
\restoresymbol{x}{rangle}
\restoresymbol{x}{hookrightarrow}
\restoresymbol{x}{hookleftarrow}
\restoresymbol{x}{Asterisk}
\ifpdf
\usepackage{pdfcolmk}
\fi
\usepackage{mdframed}
%% Force-Import aus MnSymbol
\DeclareFontFamily{U}{MnSymbolA}{}
\DeclareFontShape{U}{MnSymbolA}{m}{n}{
<-6> MnSymbolA5
<6-7> MnSymbolA6
<7-8> MnSymbolA7
<8-9> MnSymbolA8
<9-10> MnSymbolA9
<10-12> MnSymbolA10
<12-> MnSymbolA12
}{}
\DeclareFontShape{U}{MnSymbolA}{b}{n}{
<-6> MnSymbolA-Bold5
<6-7> MnSymbolA-Bold6
<7-8> MnSymbolA-Bold7
<8-9> MnSymbolA-Bold8
<9-10> MnSymbolA-Bold9
<10-12> MnSymbolA-Bold10
<12-> MnSymbolA-Bold12
}{}
\DeclareSymbolFont{MnSyA}{U}{MnSymbolA}{m}{n}
\DeclareMathSymbol{\lcirclearrowright}{\mathrel}{MnSyA}{252}
\DeclareMathSymbol{\lcirclearrowdown}{\mathrel}{MnSyA}{255}
\DeclareMathSymbol{\rcirclearrowleft}{\mathrel}{MnSyA}{250}
\DeclareMathSymbol{\rcirclearrowdown}{\mathrel}{MnSyA}{251}
\DeclareFontFamily{U}{MnSymbolC}{}
\DeclareSymbolFont{MnSyC}{U}{MnSymbolC}{m}{n}
\DeclareFontShape{U}{MnSymbolC}{m}{n}{
<-6> MnSymbolC5
<6-7> MnSymbolC6
<7-8> MnSymbolC7
<8-9> MnSymbolC8
<9-10> MnSymbolC9
<10-12> MnSymbolC10
<12-> MnSymbolC12%
}{}
\DeclareMathSymbol{\powerset}{\mathord}{MnSyC}{180}
%% ********************************************************************************
%% FILE: src/setup-parameters.tex
%% ********************************************************************************
\def\boolwahr{true}
\def\boolfalsch{false}
\def\boolleer{}
\let\documenttwosided\boolfalsch
\let\boolinappendix\boolfalsch
\let\boolinmdframed\boolfalsch
\let\eqtagset\boolfalsch
\let\eqtaglabel\boolleer
\let\eqtagsymb\boolleer
\newcount\bufferctr
\newcount\bufferreplace
\newlength\rtab
\newlength\gesamtlinkerRand
\newlength\gesamtrechterRand
\newlength\ownspaceabovethm
\newlength\ownspacebelowthm
\setlength{\rtab}{0.025\textwidth}
\setlength{\ownspaceabovethm}{0.5\baselineskip}
\setlength{\ownspacebelowthm}{0.5\baselineskip}
\setlength{\gesamtlinkerRand}{0pt}
\setlength{\gesamtrechterRand}{0pt}
\def\secnumberingpt{$\cdot$}
\def\secnumberingseppt{.}
\def\subsecnumberingseppt{}
\def\thmnumberingpt{$\cdot$}
\def\thmnumberingseppt{}
\def\thmForceSepPt{.}
\definecolor{leer}{gray}{1}
\definecolor{hellgrau}{gray}{0.85}
\definecolor{dunkelgrau}{gray}{0.5}
\definecolor{maroon}{rgb}{0.6901961,0.1882353,0.3764706}
\definecolor{dunkelgruen}{rgb}{0.015625,0.363281,0.109375}
\definecolor{dunkelrot}{rgb}{0.5450980392,0,0}
\definecolor{dunkelblau}{rgb}{0,0,0.5450980392}
\definecolor{blau}{rgb}{0,0,1}
\definecolor{newresult}{rgb}{0.6,0.6,0.6}
\definecolor{improvedresult}{rgb}{0.9,0.9,0.9}
\definecolor{hervorheben}{rgb}{0,0.9,0.7}
\definecolor{starkesblau}{rgb}{0.1019607843,0.3176470588,0.8156862745}
\definecolor{achtung}{rgb}{1,0.5,0.5}
\definecolor{frage}{rgb}{0.5,1,0.5}
\definecolor{schreibweise}{rgb}{0,0.7,0.9}
\definecolor{axiom}{rgb}{0,0.3,0.3}
%% ********************************************************************************
%% FILE: src/setup-macros.tex
%% ********************************************************************************
%% ****************************************************************
%% TEX:
%% ****************************************************************
\def\let@name#1#2{\expandafter\let\csname #1\expandafter\endcsname\csname #2\endcsname\relax}
\DeclareRobustCommand\crfamily{\fontfamily{ccr}\selectfont}
\DeclareTextFontCommand{\textcr}{\crfamily}
\def\nichtzeigen#1{\phantom{#1}}
%% ****************************************************************
%% SPACING:
%% ****************************************************************
\def\ifthenelseleer#1#2#3{\ifthenelse{\equal{#1}{}}{#2}{#1#3}}
\def\bedingtesspaceexpand#1#2#3{\ifthenelseleer{\csname #1\endcsname}{#3}{#2#3}}
\def\voritemise{\leavevmode\nvraum{1}}
\def\hraum{\null\hfill\null}
\def\vraum{\null\vfill\null}
\def\nvraum{\@ifnextchar\bgroup{\nvraum@c}{\nvraum@bes}}
\def\nvraum@c#1{\vspace*{-#1\baselineskip}}
\def\nvraum@bes{\vspace*{-\baselineskip}}
\def\erlaubeplatz{\relax\ifmmode\else\@\xspace\fi}
\def\entferneplatz{\relax\ifmmode\else\expandafter\@gobble\fi}
%% ****************************************************************
%% TAGS / BEZEICHNUNGEN / LABELLING:
%% ****************************************************************
\def\send@toaux#1{\@bsphack\protected@write\@auxout{}{\string#1}\@esphack}
%% \rlabel{LABEL}[CTR]{CREF-SHORT}{CREF-LONG}{DISPLAYTEXT}
\def\rlabel#1[#2]#3#4#5{#5\rlabel@aux{#1}[#2]{#3}{#4}{#5}}
\def\rlabel@aux#1[#2]#3#4#5{%
\send@toaux{\newlabel{#1}{{\@currentlabel}{\thepage}{{\unexpanded{#5}}}{#2.\csname the#2\endcsname}{}}}\relax%
}
%% \tag@rawscheme{CREF-SHORT}{CREF-LONG}[CTR]{LEFT-BRKT}{RIGHT-BRKT} [LABEL]{DISPLAYTEXT}
\def\tag@rawscheme#1#2[#3]#4#5{\@ifnextchar[{\tag@rawscheme@{#1}{#2}[#3]{#4}{#5}}{\tag@rawscheme@{#1}{#2}[#3]{#4}{#5}[*]}}
\def\tag@rawscheme@#1#2[#3]#4#5[#6]{\@ifnextchar\bgroup{\tag@rawscheme@@{#1}{#2}[#3]{#4}{#5}[#6]}{\tag@rawscheme@@{#1}{#2}[#3]{#4}{#5}[#6]{}}}
\def\tag@rawscheme@@#1#2[#3]#4#5[#6]#7{%
\ifthenelse{\equal{#6}{*}}{%
\ifthenelse{\equal{#7}{\boolleer}}{\refstepcounter{#3}#4\csname the#3\endcsname#5}{#4#7#5}%
}{%
\refstepcounter{#3}#4%
\ifthenelse{\equal{#7}{\boolleer}}{\rlabel{#6}[#3]{#1}{#2}{\csname the#3\endcsname}}{\rlabel{#6}[#3]{#1}{#2}{#7}}%
#5%
}%
}
%% \tag@scheme{CREF-SHORT}{CREF-LONG}[CTR] [LABEL]{DISPLAYTEXT}
\def\tag@scheme#1#2[#3]{\tag@rawscheme{#1}{#2}[#3]{\upshape(}{\upshape)}}
%% \eqtag[LABEL]{DISPLAYTEXT}
\def\eqtag@post#1{\makebox[0pt][r]{#1}}
\def\eqtag@pre{\tag@scheme{Eq}{Equation}[Xe]}
\def\eqtag{\@ifnextchar[{\eqtag@}{\eqtag@[*]}}
\def\eqtag@[#1]{\@ifnextchar\bgroup{\eqtag@@[#1]}{\eqtag@@[#1]{}}}
\def\eqtag@@[#1]#2{\eqtag@post{\eqtag@pre[#1]{#2}}}
\def\eqcref#1{\text{(\ref{#1})}}
\def\ptcref#1{\ref{#1}}
\def\punktlabel#1{\label{it:#1:\beweislabel}}
\def\punktcref#1{\eqcref{it:#1:\beweislabel}}
\def\crefit#1#2{\cref{#1}~\eqcref{it:#2:#1}}
\def\Crefit#1#2{\Cref{#1}~\eqcref{it:#2:#1}}
%% UNDER/OVERSET BEFEHLE
\def\opfromto[#1]_#2^#3{\underset{#2}{\overset{#3}{#1}}}
\def\textoverset#1#2{\overset{\text{#1}}{#2}}
\def\textunderset#1#2{\underset{#2}{\text{#1}}}
\def\crefoverset#1#2{\textoverset{\cref{#1}}{#2}}
\def\Crefoverset#1#2{\textoverset{\Cref{#1}}{#2}}
\def\crefunderset#1#2{\textunderset{#2}{\cref{#1}}}
\def\Crefunderset#1#2{\textunderset{#2}{\Cref{#1}}}
\def\eqcrefoverset#1#2{\textoverset{\eqcref{#1}}{#2}}
\def\eqcrefunderset#1#2{\textunderset{#2}{\eqcref{#1}}}
\def\mathclap#1{#1}
\def\oberunterset#1{\@ifnextchar^{\oberunterset@oben{#1}}{\oberunterset@unten{#1}}}
\def\oberunterset@oben#1^#2_#3{\underset{\mathclap{#3}}{\overset{\mathclap{#2}}{#1}}}
\def\oberunterset@unten#1_#2^#3{\underset{\mathclap{#2}}{\overset{\mathclap{#3}}{#1}}}
\def\breitunderbrace#1_#2{\underbrace{#1}_{\mathclap{#2}}}
\def\breitoverbrace#1^#2{\overbrace{#1}^{\mathclap{#2}}}
\def\breitunderbracket#1_#2{\underbracket{#1}_{\mathclap{#2}}}
\def\breitoverbracket#1^#2{\overbracket{#1}^{\mathclap{#2}}}
\def\generatenestedsecnumbering#1#2#3{%
\expandafter\gdef\csname thelong#3\endcsname{%
\expandafter\csname the#2\endcsname%
\secnumberingpt%
\expandafter\csname #1\endcsname{#3}%
}%
\expandafter\gdef\csname theshort#3\endcsname{%
\expandafter\csname #1\endcsname{#3}%
}%
}
\def\generatenestedthmnumbering#1#2#3{%
\expandafter\gdef\csname the#3\endcsname{%
\expandafter\csname the#2\endcsname%
\thmnumberingpt%
\expandafter\csname #1\endcsname{#3}%
}%
\expandafter\gdef\csname theshort#3\endcsname{%
\expandafter\csname #1\endcsname{#3}%
}%
}
%% ****************************************************************
%% ALLG. MACROS:
%% ****************************************************************
\def\+#1{\addtocounter{#1}{1}}
\def\setcounternach#1#2{\setcounter{#1}{#2}\addtocounter{#1}{-1}}
\def\textsubscript#1{${}_{\textup{#1}}$}
\def\rome#1{\overline{\underline{#1}}}
\def\textTODO{\text{[{\large\textcolor{red}{More work needed!}}]}}
\def\hlineEIGENpt{\hdashline[0.5pt/5pt]}
\def\clineEIGENpt#1{\cdashline{#1}[0.5pt/5pt]}
\def\forcepunkt#1{#1\IfEndWith{#1}{.}{}{.}}
\def\lateinabkuerzung#1#2{%
\expandafter\gdef\csname #1\endcsname{\emph{#2}\@ifnextchar.{\entferneplatz}{\erlaubeplatz}}
}
\def\deutscheabkuerzung#1#2{%
\expandafter\gdef\csname #1\endcsname{{#2}\@ifnextchar.{\entferneplatz}{\erlaubeplatz}}
}
%% ****************************************************************
%% MATHE
%% ****************************************************************
\def\matrix#1{\left(\begin{array}{#1}}
\def\endmatrix{\end{array}\right)}
\def\smatrix{\left(\begin{smallmatrix}}
\def\endsmatrix{\end{smallmatrix}\right)}
\def\multiargrekursiverbefehl#1#2#3#4#5#6#7#8{%
\expandafter\gdef\csname#1\endcsname #2##1#4{\csname #1@anfang\endcsname##1#3\egroup}
\expandafter\def\csname #1@anfang\endcsname##1#3{#5##1\@ifnextchar\egroup{\csname #1@ende\endcsname}{#7\csname #1@mitte\endcsname}}
\expandafter\def\csname #1@mitte\endcsname##1#3{#6##1\@ifnextchar\egroup{\csname #1@ende\endcsname}{#7\csname #1@mitte\endcsname}}
\expandafter\def\csname #1@ende\endcsname##1{#8}
}
\multiargrekursiverbefehl{svektor}{[}{;}{]}{\begin{smatrix}}{}{\\}{\\\end{smatrix}}
\multiargrekursiverbefehl{vektor}{[}{;}{]}{\begin{matrix}{c}}{}{\\}{\\\end{matrix}}
\multiargrekursiverbefehl{vektorzeile}{}{,}{;}{}{&}{}{}
\multiargrekursiverbefehl{matlabmatrix}{[}{;}{]}{\begin{smatrix}\vektorzeile}{\vektorzeile}{;\\}{;\end{smatrix}}
\def\cases[#1]#2{\left\{\begin{array}[#1]{#2}}
\def\endcases{\end{array}\right.}
\def\BeweisRichtung[#1]{\@ifnextchar\bgroup{\@BeweisRichtung@c[#1]}{\@BeweisRichtung@bes[#1]}}
\def\@BeweisRichtung@bes[#1]{{\bfseries(#1).~}}
\def\@BeweisRichtung@c[#1]#2#3{{\bfseries(#2#1#3).~}}
\def\erzeugeBeweisRichtungBefehle#1#2{
\expandafter\gdef\csname #1text\endcsname##1##2{\BeweisRichtung[#2]{##1}{##2}}
\expandafter\gdef\csname #1\endcsname{%
\@ifnextchar\bgroup{\csname #1@\endcsname}{\csname #1text\endcsname{}{}}%
}
\expandafter\gdef\csname #1@\endcsname##1##2{%
\csname #1text\endcsname{\punktcref{##1}}{\punktcref{##2}}%
}
}
\erzeugeBeweisRichtungBefehle{hinRichtung}{$\Longrightarrow$}
\erzeugeBeweisRichtungBefehle{herRichtung}{$\Longleftarrow$}
\erzeugeBeweisRichtungBefehle{hinherRichtung}{$\Longleftrightarrow$}
\def\cal#1{\mathcal{#1}}
\def\brkt#1{\langle{}#1{}\rangle}
\def\mathfrak#1{\mbox{\usefont{U}{euf}{m}{n}#1}}
\def\kurs#1{\textit{#1}}
\def\rectangleblack{\text{\RectangleBold}}
\def\rectanglewhite{\text{\Rectangle}}
\def\squareblack{\blacksquare}
\def\squarewhite{\Box}
%% ********************************************************************************
%% FILE: src/setup-environments.tex
%% ********************************************************************************
%% **********************************************************************
%% CLEVEREF: ************************************************************
\def\crefname@full#1#2#3{\crefname{#1}{#2}{#3}\Crefname{#1}{#2}{#3}}
\crefname@full{chapter}{Kapitel}{Kapitel}
\crefname@full{section}{Abschnitt}{Abschnitte}
\crefname@full{figure}{Fig.}{Fig.}
\crefname@full{subfigure}{Fig.}{Fig.}
\crefname@full{proof}{Beweis}{Beweise}
\crefname@full{thm}{Theorem}{Theoreme}
\crefname@full{satz}{Satz}{Sätze}
\crefname@full{claim}{Behauptung}{Behauptungen}
\crefname@full{lemm}{Lemma}{Lemmata}
\crefname@full{cor}{Korollar}{Korollarien}
\crefname@full{folg}{Folgerung}{Folgerungen}
\crefname@full{prop}{Proposition}{Propositionen}
\crefname@full{defn}{Definition}{Definitionen}
\crefname@full{conv}{Konvention}{Konventionen}
\crefname@full{fact}{Fakt}{Fakten}
\crefname@full{rem}{Bemerkung}{Bemerkungen}
\crefname@full{qstn}{Frage}{Fragen}
\crefname@full{e.g.}{Beipsiel}{Beipsiele}
%% ****************************************************************
%% THEOREME:
%% ****************************************************************
\def\qedEIGEN#1{\@ifnextchar[{\qedEIGEN@c{#1}}{\qedEIGEN@bes{#1}}}%]
\def\qedEIGEN@bes#1{%
\parfillskip=0pt% % so \par doesnt push \square to left
\widowpenalty=10000% % so we dont break the page before \square
\displaywidowpenalty=10000% % ditto
\finalhyphendemerits=0% % TeXbook exercise 14.32
\leavevmode% % \nobreak means lines not pages
\unskip% % remove previous space or glue
\nobreak% % dont break lines
\hfil% % ragged right if we spill over
\penalty50% % discouragement to do so
\hskip.2em% % ensure some space
\null% % anchor following \hfill
\hfill% % push \square to right
#1% % the end-of-proof mark
\par%
}
\def\qedEIGEN@c#1[#2]{%
\parfillskip=0pt% % so \par doesnt push \square to left
\widowpenalty=10000% % so we dont break the page before \square
\displaywidowpenalty=10000% % ditto
\finalhyphendemerits=0% % TeXbook exercise 14.32
\leavevmode% % \nobreak means lines not pages
\unskip% % remove previous space or glue
\nobreak% % dont break lines
\hfil% % ragged right if we spill over
\penalty50% % discouragement to do so
\hskip.2em% % ensure some space
\null% % anchor following \hfill
\hfill% % push \square to right
{#1~{\smaller\bfseries\upshape (#2)}}%
\par%
}
\def\qedVARIANT#1#2{
\expandafter\def\csname ennde#1Sign\endcsname{#2}
\expandafter\def\csname ennde#1\endcsname{\@ifnextchar[{\qedEIGEN@c{#2}}{\qedEIGEN@bes{#2}}} %]
}
\qedVARIANT{OfProof}{$\squareblack$}
\qedVARIANT{OfWork}{\rectangleblack}
\qedVARIANT{OfSomething}{$\dashv$}
\qedVARIANT{OnNeutral}{$\lozenge$} % \lozenge \bigcirc \blacklozenge
\def\qedsymbol{\enndeOfProofSign}
\def\proofSymbol{\enndeOfProofSign}
\def\ra@pretheoremwork{
\setlength{\theorempreskipamount}{\ownspaceabovethm}
}
\def\rathmtransfer#1#2{
\expandafter\def\csname #2\endcsname{\csname #1\endcsname}
\expandafter\def\csname end#2\endcsname{\csname end#1\endcsname}
}
\def\ranewthm#1#2#3[#4]{
%% FOR \BEGIN{THM}
\theoremstyle{\current@theoremstyle}
\theoremseparator{\current@theoremseparator}
\theoremprework{\ra@pretheoremwork}
\@ifundefined{#1@basic}{\newtheorem{#1@basic}[#4]{#2}}{\renewtheorem{#1@basic}[#4]{#2}}
%% FOR \BEGIN{THM}[...]
\theoremstyle{\current@theoremstyle}
\theoremseparator{\thmForceSepPt}
\theoremprework{\ra@pretheoremwork}
\@ifundefined{#1@withName}{\newtheorem{#1@withName}[#4]{#2}}{\renewtheorem{#1@withName}[#4]{#2}}
%% FOR \BEGIN{THM*}
\theoremstyle{nonumberplain}
\theoremseparator{\thmForceSepPt}
\theoremprework{\ra@pretheoremwork}
\@ifundefined{#1@star@basic}{\newtheorem{#1@star@basic}[#4]{#2}}{\renewtheorem{#1@star@basic}[#4]{#2}}
%% FOR \BEGIN{THM*}[...]
\theoremstyle{nonumberplain}
\theoremseparator{\thmForceSepPt}
\theoremprework{\ra@pretheoremwork}
\@ifundefined{#1@star@withName}{\newtheorem{#1@star@withName}[#4]{#2}}{\renewtheorem{#1@star@withName}[#4]{#2}}
%% GENERATE ENVIRONMENTS:
\umbauenenv{#1}{#3}[#4]
\umbauenenv{#1@star}{#3}[#4]
%% TRANSFER *-DEFINITION
\rathmtransfer{#1@star}{#1*}
}
\def\umbauenenv#1#2[#3]{%
%% \BEGIN{THM}...
\expandafter\def\csname #1\endcsname{\relax%
\@ifnextchar[{\csname #1@\endcsname}{\csname #1@\endcsname[*]}%
}
%% \BEGIN{THM}[ANFANG]...
\expandafter\def\csname #1@\endcsname[##1]{\relax%
\@ifnextchar[{\csname #1@@\endcsname[##1]}{\csname #1@@\endcsname[##1][*]}%
}
%% \BEGIN{THM}[ANFANG][SCHLUSS]
\expandafter\def\csname #1@@\endcsname[##1][##2]{%
\ifx*##1%
\def\enndeOfBlock{\csname end#1@basic\endcsname}
\csname #1@basic\endcsname%
\else%
\def\enndeOfBlock{\csname end#1@withName\endcsname}
\csname #1@withName\endcsname[##1]%
\fi%
\def\makelabel####1{%
\gdef\beweislabel{####1}%
\label{\beweislabel}%
}%
\ifx*##2%
\def\enndeSymbol{\qedEIGEN{#2}}
\else%
\def\enndeSymbol{\qedEIGEN{#2}[##2]}
\fi
}
%% \END{THM}
\expandafter\gdef\csname end#1\endcsname{\enndeSymbol\enndeOfBlock}
}
%% NEWTHEOREM EINSTELLUNGSOPTIONEN:
%% F\"UR \theoremstyle
%% plain Emulates original LATEX defin, except uses param \theorem...skipamount.
%% break Header followed by line break.
%% change Header, Number and Text are interchanged, without a line break.
%% changebreak =change, but with a line break after Header.
%% margin Number in left margin, without a line break.
%% marginbreak =margin, but with a line break after the header.
%% nonumberplain =plain, without number.
%% nonumberbreak =break, without number.
%% empty No number, no name. Only the optional argument is typeset.
%% \theoremclass \theoremnumbering
%% \theorempreskip \theorempostkip \theoremindent
%% \theoremprework \theorempostwork
\def\current@theoremstyle{plain}
\def\current@theoremseparator{\thmnumberingseppt}
\theoremstyle{\current@theoremstyle}
\theoremseparator{\current@theoremseparator}
\theoremsymbol{}
\newtheorem{X}{X}[chapter] % for most theorems
\newtheorem{Xe}{Xe}[chapter] % for equations
\newtheorem*{Xdisplaynone}{Xdisplaynone}[chapter] % a dummy counter, that will never be displayed.
\newtheorem{Xsp}{Xsp}[chapter] % for special theorems
\generatenestedthmnumbering{arabic}{chapter}{X}
\generatenestedthmnumbering{arabic}{chapter}{Xe}
\generatenestedthmnumbering{Roman}{chapter}{Xsp}
\let\theXsp\theshortXsp
\theoremheaderfont{\upshape\bfseries}
\theorembodyfont{\slshape}
\ranewthm{thm}{Theorem}{\enndeOnNeutralSign}[X]
\ranewthm{satz}{Satz}{\enndeOnNeutralSign}[X]
\ranewthm{claim}{Behauptung}{\enndeOnNeutralSign}[X]
\ranewthm{lemm}{Lemma}{\enndeOnNeutralSign}[X]
\ranewthm{cor}{Korollar}{\enndeOnNeutralSign}[X]
\ranewthm{folg}{Folgerung}{\enndeOnNeutralSign}[X]
\ranewthm{prop}{Proposition}{\enndeOnNeutralSign}[X]
\theorembodyfont{\upshape}
\ranewthm{defn}{Definition}{\enndeOnNeutralSign}[X]
\ranewthm{conv}{Konvention}{\enndeOnNeutralSign}[X]
\ranewthm{e.g.}{Beipsiel}{\enndeOnNeutralSign}[X]
\ranewthm{fact}{Fakt}{\enndeOnNeutralSign}[X]
\ranewthm{rem}{Bemerkung}{\enndeOnNeutralSign}[X]
\ranewthm{qstn}{Frage}{\enndeOnNeutralSign}[X]
\theoremheaderfont{\itshape\bfseries}
\theorembodyfont{\upshape}
\ranewthm{proof@tmp}{Beweis}{\enndeOfProofSign}[Xdisplaynone]
\rathmtransfer{proof@tmp*}{proof}
\def\behauptungbeleg@claim{%
\iflanguage{british}{Claim}{%
\iflanguage{english}{Claim}{%
\iflanguage{ngerman}{Behauptung}{%
\iflanguage{russian}{Утверждение}{%
Claim%
}}}}%
}
\def\behauptungbeleg@pf@kurz{%
\iflanguage{british}{Pf}{%
\iflanguage{english}{Pf}{%
\iflanguage{ngerman}{Bew}{%
\iflanguage{russian}{Доказательство}{%
Pf%
}}}}%
}
\def\behauptungbeleg{\@ifnextchar\bgroup{\behauptungbeleg@c}{\behauptungbeleg@bes}}
\def\behauptungbeleg@c#1{\item[{\bfseries \behauptungbeleg@claim\erlaubeplatz #1.}]}
\def\behauptungbeleg@bes{\item[{\bfseries \behauptungbeleg@claim.}]}
\def\belegbehauptung{\item[{\bfseries\itshape\behauptungbeleg@pf@kurz.}]}
%% ****************************************************************
%% ALTE UMGEBUNGEN:
%% ****************************************************************
\newcolumntype{\RECHTS}[1]{>{\raggedleft}p{#1}}
\newcolumntype{\LINKS}[1]{>{\raggedright}p{#1}}
\newcolumntype{m}{>{$}l<{$}}
\newcolumntype{C}{>{$}c<{$}}
\newcolumntype{L}{>{$}l<{$}}
\newcolumntype{R}{>{$}r<{$}}
\newcolumntype{0}{@{\hspace{0pt}}}
\newcolumntype{\LINKSRAND}{@{\hspace{\@totalleftmargin}}}
\newcolumntype{h}{@{\extracolsep{\fill}}}
\newcolumntype{i}{>{\itshape}}
\newcolumntype{t}{@{\hspace{\tabcolsep}}}
\newcolumntype{q}{@{\hspace{1em}}}
\newcolumntype{n}{@{\hspace{-\tabcolsep}}}
\newcolumntype{M}[2]{%
>{\begin{minipage}{#2}\begin{math}}%
{#1}%
<{\end{math}\end{minipage}}%
}
\newcolumntype{T}[2]{%
>{\begin{minipage}{#2}}%
{#1}%
<{\end{minipage}}%
}
\setlength{\LTpre}{\baselineskip}
\setlength{\LTpost}{0pt}
\def\center{\centering}
\def\endcenter{}
\def\punkteumgebung@genbefehl#1#2#3{
\punkteumgebung@genbefehl@{#1}{#2}{#3}{}{}
\punkteumgebung@genbefehl@{multi#1}{#2}{#3}{
\setlength{\columnsep}{10pt}%
\setlength{\columnseprule}{0pt}%
\begin{multicols}{\thecolumnanzahl}%
}{\end{multicols}\nvraum{1}}
}
\def\punkteumgebung@genbefehl@#1#2#3#4#5{
\expandafter\gdef\csname #1\endcsname{
\@ifnextchar\bgroup{\csname #1@c\endcsname}{\csname #1@bes\endcsname}
}%]
\expandafter\def\csname #1@c\endcsname##1{
\@ifnextchar[{\csname #1@c@\endcsname{##1}}{\csname #1@c@\endcsname{##1}[\z@]}
}%]
\expandafter\def\csname #1@c@\endcsname##1[##2]{
\@ifnextchar[{\csname #1@c@@\endcsname{##1}[##2]}{\csname #1@c@@\endcsname{##1}[##2][\z@]}
}%]
\expandafter\def\csname #1@c@@\endcsname##1[##2][##3]{
\let\alterlinkerRand\gesamtlinkerRand
\let\alterrechterRand\gesamtrechterRand
\addtolength{\gesamtlinkerRand}{##2}
\addtolength{\gesamtrechterRand}{##3}
\advance\linewidth -##2%
\advance\linewidth -##3%
\advance\@totalleftmargin ##2%
\parshape\@ne \@totalleftmargin\linewidth%
#4
\begin{#2}[\upshape ##1]%
\setlength{\parskip}{0.5\baselineskip}\relax%
\setlength{\topsep}{\z@}\relax%
\setlength{\partopsep}{\z@}\relax%
\setlength{\parsep}{\parskip}\relax%
\setlength{\itemsep}{#3}\relax%
\setlength{\listparindent}{\z@}\relax%
\setlength{\itemindent}{\z@}\relax%
}
\expandafter\def\csname #1@bes\endcsname{
\@ifnextchar[{\csname #1@bes@\endcsname}{\csname #1@bes@\endcsname[\z@]}
}%]
\expandafter\def\csname #1@bes@\endcsname[##1]{
\@ifnextchar[{\csname #1@bes@@\endcsname[##1]}{\csname #1@bes@@\endcsname[##1][\z@]}
}%]
\expandafter\def\csname #1@bes@@\endcsname[##1][##2]{
\let\alterlinkerRand\gesamtlinkerRand
\let\alterrechterRand\gesamtrechterRand
\addtolength{\gesamtlinkerRand}{##1}
\addtolength{\gesamtrechterRand}{##2}
\advance\linewidth -##1%
\advance\linewidth -##2%
\advance\@totalleftmargin ##1%
\parshape\@ne \@totalleftmargin\linewidth%
#4
\begin{#2}%
\setlength{\parskip}{0.5\baselineskip}\relax%
\setlength{\topsep}{\z@}\relax%
\setlength{\partopsep}{\z@}\relax%
\setlength{\parsep}{\parskip}\relax%
\setlength{\itemsep}{#3}\relax%
\setlength{\listparindent}{\z@}\relax%
\setlength{\itemindent}{\z@}\relax%
}
\expandafter\gdef\csname end#1\endcsname{%
\end{#2}#5
\setlength{\gesamtlinkerRand}{\alterlinkerRand}
\setlength{\gesamtlinkerRand}{\alterrechterRand}
}
}
\def\ritempunkt{{\Large\textbullet}} % \textbullet, $\sqbullet$, $\blacktriangleright$
\setdefaultitem{\ritempunkt}{\ritempunkt}{\ritempunkt}{\ritempunkt}
\punkteumgebung@genbefehl{itemise}{compactitem}{\parskip}{}{}
\punkteumgebung@genbefehl{kompaktitem}{compactitem}{\z@}{}{}
\punkteumgebung@genbefehl{enumerate}{compactenum}{\parskip}{}{}
\punkteumgebung@genbefehl{kompaktenum}{compactenum}{\z@}{}{}
\let\ALTthebibliography\thebibliography
\renewenvironment{thebibliography}[1]{%
\begin{ALTthebibliography}{#1}
\addcontentsline{toc}{part}{\bibname}
}{%
\end{ALTthebibliography}
}
%% ****************************************************************
%% NEUE UMGEBUNGEN:
%% ****************************************************************
\def\matrix#1{\left(\begin{array}[mc]{#1}}
\def\endmatrix{\end{array}\right)}
\def\smatrix{\left(\begin{smallmatrix}}
\def\endsmatrix{\end{smallmatrix}\right)}
\def\vector{\begin{matrix}{c}}
\def\endvector{\end{matrix}}
\def\svector{\begin{smatrix}}
\def\endsvector{\end{smatrix}}
\def\multiargrekursiverbefehl#1#2#3#4#5#6#7#8{%
\expandafter\gdef\csname#1\endcsname #2##1#4{\csname #1@anfang\endcsname##1#3\egroup}
\expandafter\def\csname #1@anfang\endcsname##1#3{#5##1\@ifnextchar\egroup{\csname #1@ende\endcsname}{#7\csname #1@mitte\endcsname}}
\expandafter\def\csname #1@mitte\endcsname##1#3{#6##1\@ifnextchar\egroup{\csname #1@ende\endcsname}{#7\csname #1@mitte\endcsname}}
\expandafter\def\csname #1@ende\endcsname##1{#8}
}
\multiargrekursiverbefehl{svektor}{[}{;}{]}{\begin{smatrix}}{}{\\}{\\\end{smatrix}}
\multiargrekursiverbefehl{vektor}{[}{;}{]}{\begin{matrix}{c}}{}{\\}{\\\end{matrix}}
\multiargrekursiverbefehl{vektorzeile}{}{,}{;}{}{&}{}{}
\multiargrekursiverbefehl{matlabmatrix}{[}{;}{]}{\begin{smatrix}\vektorzeile}{\vektorzeile}{;\\}{;\end{smatrix}}
\def\underbracenodisplay#1{%
\mathop{\vtop{\m@th\ialign{##\crcr
$\hfil\displaystyle{#1}\hfil$\crcr
\noalign{\kern3\p@\nointerlineskip}%
\upbracefill\crcr\noalign{\kern3\p@}}}}\limits%
}
\def\mathe[#1]#2{%
\ifthenelse{\equal{\boolinmdframed}{\boolwahr}}{}{\begin{escapeeinzug}}
\noindent%
\let\eqtagset\boolfalsch
\let\eqtaglabel\boolleer
\let\eqtagsymb\boolleer
\let\alteqtag\eqtag
\def\eqtag{\@ifnextchar[{\eqtag@loc@}{\eqtag@loc@[*]}}%
\def\eqtag@loc@[##1]{\@ifnextchar\bgroup{\eqtag@loc@@[##1]}{\eqtag@loc@@[##1]{}}}%
\def\eqtag@loc@@[##1]##2{%
\gdef\eqtagset{\boolwahr}
\gdef\eqtaglabel{##1}
\gdef\eqtagsymb{##2}
}%
\def\verticalalign{}%
\IfBeginWith{#1}{t}{\def\verticalalign{t}}{}%
\IfBeginWith{#1}{m}{\def\verticalalign{c}}{}%
\IfBeginWith{#1}{b}{\def\verticalalign{b}}{}%
\def\horizontalalign{\null\hfill\null}%
\IfEndWith{#1}{l}{}{\null\hfill\null}%
\IfEndWith{#1}{r}{\def\horizontalalign{}}{}%
\begin{math}
\begin{array}[\verticalalign]{0#2}%
}
\def\endmathe{%
\end{array}
\end{math}\horizontalalign%
\let\eqtag\alteqtag
\ifthenelse{\equal{\eqtagset}{\boolwahr}}{\eqtag[\eqtaglabel]{\eqtagsymb}}{}
\ifthenelse{\equal{\boolinmdframed}{\boolwahr}}{}{\end{escapeeinzug}}%
}
\def\longmathe[#1]#2{\relax
\let\altarraystretch\arraystretch
\renewcommand\arraystretch{1.2}\relax
\begin{longtable}[#1]{\LINKSRAND #2}
}
\def\endlongmathe{
\end{longtable}
\renewcommand\arraystretch{\altarraystretch}
}
\def\einzug{\@ifnextchar[{\indents@}{\indents@[\z@]}}%]
\def\indents@[#1]{\@ifnextchar[{\indents@@[#1]}{\indents@@[#1][\z@]}}%]
\def\indents@@[#1][#2]{%
\begin{list}{}{\relax
\setlength{\topsep}{\z@}\relax
\setlength{\partopsep}{\z@}\relax
\setlength{\parsep}{\parskip}\relax
\setlength{\listparindent}{\z@}\relax
\setlength{\itemindent}{\z@}\relax
\setlength{\leftmargin}{#1}\relax
\setlength{\rightmargin}{#2}\relax
\let\alterlinkerRand\gesamtlinkerRand
\let\alterrechterRand\gesamtrechterRand
\addtolength{\gesamtlinkerRand}{#1}
\addtolength{\gesamtrechterRand}{#2}
}\relax
\item[]\relax
}
\def\endeinzug{%
\setlength{\gesamtlinkerRand}{\alterlinkerRand}
\setlength{\gesamtlinkerRand}{\alterrechterRand}
\end{list}%
}
\def\escapeeinzug{\begin{einzug}[-\gesamtlinkerRand][-\gesamtrechterRand]}
\def\endescapeeinzug{\end{einzug}}
\def\programmiercode{
\modulolinenumbers[1]
\begin{einzug}[\rtab][\rtab]%
\begin{linenumbers}%
\fontfamily{cmtt}\fontseries{m}\fontshape{u}\selectfont%
\setlength{\parskip}{1\baselineskip}%
\setlength{\parindent}{0pt}%
}
\def\endprogrammiercode{
\end{linenumbers}
\end{einzug}
}
\def\schattiertebox@genbefehl#1#2#3{
\expandafter\gdef\csname #1\endcsname{%
\@ifnextchar[{\csname #1@args\endcsname}{\csname #1@args\endcsname[#3]}%]%
}
\expandafter\def\csname #1@args\endcsname[##1]{%
\@ifnextchar[{\csname #1@args@l\endcsname[##1]}{\csname #1@args@n\endcsname[##1]}%]%
}
\expandafter\def\csname #1@args@l\endcsname[##1][##2]{%
\@ifnextchar[{\csname #1@args@l@r\endcsname[##1][##2]}{\csname #1@args@l@n\endcsname[##1][##2]}%]%
}
\expandafter\def\csname #1@args@n\endcsname[##1]{%
\let\boolinmdframed\boolwahr
\begin{mdframed}[#2leftmargin=0,rightmargin=0,outermargin=0,innermargin=0,##1]
}
\expandafter\def\csname #1@args@l@n\endcsname[##1][##2]{%
\let\boolinmdframed\boolwahr
\begin{mdframed}[#2leftmargin=##2/2,rightmargin=##2/2,outermargin=##2/2,innermargin=##2/2,##1]
}
\expandafter\def\csname #1@args@l@r\endcsname[##1][##2][##3]{%
\let\boolinmdframed\boolwahr
\begin{mdframed}[#2leftmargin=##2,rightmargin=##3,outermargin=##2,innermargin=##3,##1]
}
\expandafter\gdef\csname end#1\endcsname{%
\end{mdframed}
\let\boolinmdframed\boolfalsch
}
}
\schattiertebox@genbefehl{schattiertebox}{
splittopskip=0,%
splitbottomskip=0,%
frametitleaboveskip=0,%
frametitlebelowskip=0,%
skipabove=1\baselineskip,%
skipbelow=1\baselineskip,%
linewidth=2pt,%
linecolor=black,%
roundcorner=4pt,%
}{
backgroundcolor=leer,%
nobreak=true,%
}
\schattiertebox@genbefehl{schattierteboxdunn}{
splittopskip=0,%
splitbottomskip=0,%
frametitleaboveskip=0,%
frametitlebelowskip=0,%
skipabove=1\baselineskip,%
skipbelow=1\baselineskip,%
linewidth=1pt,%
linecolor=black,%
roundcorner=2pt,%
}{
backgroundcolor=leer,%
nobreak=true,%
}
\def\algorithm{\schattiertebox[backgroundcolor=hellgrau,nobreak=false]}
\def\endalgorithm{\endschattiertebox}
\def\tikzsetzenode#1{%
\tikz[remember picture,baseline,overlay]{\node #1;}%
}
\def\tikzsetzepfeil#1{%
\begin{tikzpicture}[remember picture,overlay,>=latex]%
\draw #1;%
\end{tikzpicture}%
}
\def\tikzsetzeoverlay#1{%
\begin{tikzpicture}[remember picture,overlay,>=latex]%
#1%
\end{tikzpicture}%
}
\def\tikzsetzekreise[#1]#2#3{%
\tikzsetzepfeil{%
[rounded corners,#1]%
([shift={(-\tabcolsep,0.75\baselineskip)}]#2)%
rectangle%
([shift={(\tabcolsep,-0.5\baselineskip)}]#3)
}%
}
\tikzset{
>=stealth,
auto,
node distance=1cm,
thick,
main node/.style={
circle,draw,font=\sffamily\Large\bfseries,minimum size=0pt
},
state/.style={minimum size=0pt}
loop above right/.style={loop,out=30,in=60,distance=0.5cm},
loop above left/.style={above left,out=150,in=120,loop},
loop below right/.style={below right,out=330,in=300,loop},
loop below left/.style={below left,out=240,in=210,loop},
itria/.style={
draw,dashed,shape border uses incircle,
isosceles triangle,shape border rotate=90,yshift=-1.45cm
},
rtria/.style={
draw,dashed,shape border uses incircle,
isosceles triangle,isosceles triangle apex angle=90,
shape border rotate=-45,yshift=0.2cm,xshift=0.5cm
},
ritria/.style={
draw,dashed,shape border uses incircle,
isosceles triangle,isosceles triangle apex angle=110,
shape border rotate=-55,yshift=0.1cm
},
litria/.style={
draw,dashed,shape border uses incircle,
isosceles triangle,isosceles triangle apex angle=110,
shape border rotate=235,yshift=0.1cm
}
}
%% ********************************************************************************
%% FILE: src/setup-layout.tex
%% ********************************************************************************
\pagestyle{fancyplain}
\@ifundefined{setcitestyle}{%
%% do nothing
}{%
\setcitestyle{numeric-comp,open={[},close={]}}
}
\def\crefpairconjunction{ und }
\def\crefmiddleconjunction{, }
\def\creflastconjunction{, und }
\raggedbottom %% <- pushes footers up
\sloppy
\def\headrulewidth{0pt}
\def\footrulewidth{0pt}
\setlength{\columnsep}{20pt}
\setlength{\columnseprule}{1pt}
\setlength{\headheight}{11pt}
\setlength{\partopsep}{0pt}
\setlength{\topsep}{\baselineskip}
\setlength{\topskip}{0.5\baselineskip}
\setlength{\footskip}{-1\baselineskip}
\setlength{\maxdepth}{0pt}
\renewcommand{\baselinestretch}{1}
\renewcommand{\arraystretch}{1}
\setcounter{LTchunksize}{\infty}
\setlength{\abovedisplayskip}{0pt}
\setlength{\parskip}{1\baselineskip}
\def\firstparagraph{\noindent}
\def\continueparagraph{\noindent}
\hypersetup{
hidelinks=true,
}
\@addtoreset{chapter}{part} %% nötig für Hyperref.
\def\partfont{\documentfont\fontseries{bx}\Huge\selectfont}
\def\chapterfont{\documentfont\fontseries{bx}\huge\selectfont}
\def\sectionfont{\documentfont\fontseries{bx}\Large\selectfont}
\def\subsectionfont{\documentfont\fontseries{bx}\large\selectfont}
\def\thepart{\Roman{part}}
\generatenestedsecnumbering{arabic}{part}{chapter}
\generatenestedsecnumbering{arabic}{chapter}{section}
\generatenestedsecnumbering{arabic}{section}{subsection}
\generatenestedsecnumbering{arabic}{subsection}{subsubsection}
\def\theunitnamepart{\thepart}
\def\theunitnamechapter{\theshortchapter}
\def\theunitnamesection{\thelongsection}
\def\theunitnamesubsection{\thelongsubsection}
\def\theunitnamesubsubsection{\thelongsubsubsection}
\def\partname{Teil\erlaubeplatz}
\def\chaptername{Kapitel\erlaubeplatz}
\def\sectionname{\S\erlaubeplatz}
\def\subsectionname{}
\def\subsubsectionname{}
\let\appendix@orig\appendix
\def\appendix{%
\appendix@orig%
\let\boolinappendix\boolwahr
\addcontentsline{toc}{part}{\appendixname}%
\addtocontents{toc}{\protect\setcounter{tocdepth}{0}}
\def\sectionname{Appendix}%
\def\theunitnamesection{\Alph{section}}%
}
\def\notappendix{%
\let\boolinappendix\boolfalse
\addtocontents{toc}{\protect\setcounter{tocdepth}{1 }}
\def\sectionname{}%
\def\theunitnamesection{\arabic{section}}%
}
%% \titlespacing{<sectionclassname>}
%% {linker einzug}{platz oberhalb}{platz unterhalb}[rechter einzug]
\titlespacing{\section}{0pt}{\baselineskip}{\baselineskip}
\titlespacing{\subsection}{0pt}{\baselineskip}{\baselineskip}
\titlespacing{\subsubsection}{0pt}{\baselineskip}{\baselineskip}
\titlespacing{\paragraph}{0pt}{0pt}{1em}
\titleformat{\part}[display]
{\normalfont\headingfont\bfseries\Huge\centering}
{%
\ifthenelse{\equal{\partname}{}}{%
\theunitnamepart%
}{%
\MakeUppercase{\partname}~\theunitnamepart%
}%
}{0pt}{%
}[\thispagestyle{empty}]
\titleformat{\chapter}[frame]
{\normalfont\headingfont\bfseries\Large}
{%
\bedingtesspaceexpand{chaptername}{~}{\theunitnamechapter}%
}{0.5em}{%
}[\thispagestyle{empty}]%\titlerule%[2pt]%
\titleformat{\section}[hang]
{\normalfont\headingfont\bfseries\flushleft\large}
{%
\bedingtesspaceexpand{sectionname}{~}{\theunitnamesection}%
}{0.5em}
{%
}
[%
\nvraum{0.25}%
]
\titleformat{\subsection}[hang]
{\normalfont\headingfont\bfseries\flushleft\large}
{%
\bedingtesspaceexpand{subsectionname}{~}{\theunitnamesubsection}%
}{0.5em}
{%
}
[%
\nvraum{0.25}%
]
\titleformat{\subsubsection}[hang]
{\normalfont\headingfont\bfseries\flushleft\large}
{%
\bedingtesspaceexpand{subsubsectionname}{~}{\theunitnamesubsubsection}%
}{0.5em}
{%
}
[%
\nvraum{0.25}%
]
\def\rafootnotectr{20}
\def\incrftnotectr#1{%
\addtocounter{#1}{1}%
\ifnum\value{#1}>\rafootnotectr\relax
\setcounter{#1}{0}%
\fi%
}
\def\footnoteref[#1]{\protected@xdef\@thefnmark{\ref{#1}}\@footnotemark}
\let\altfootnotetext\footnotetext
\def\footnotetext[#1]#2{\incrftnotectr{footnote}\altfootnotetext[\value{footnote}]{\label{#1}#2}}
\let\altfootnotemark\footnotemark
%% Undesirable solution, as the text is not hyperlinked.
\def\footnotemark[#1]{\text{\textsuperscript{\getrefnumber{#1}}}}
\DefineFNsymbols*{custom}{abcdefghijklmnopqrstuvwxyz}
\setfnsymbol{custom}
\def\footnotelayout{\documentfont\scriptsize}
\def\thefootnote{\fnsymbol{footnote}}
\def\kopfzeileleer{
\lhead[]{}
\chead[]{}
\rhead[]{}
\lfoot[]{}
\cfoot[]{}
\rfoot[]{}
}
\def\kopfzeiledefault{
\lhead[]{}
\lhead[]{}
\chead[]{}
\rhead[]{}
\lfoot[]{}
\cfoot{\footnotesize\thepage}
\rfoot[]{}
}
\DeclareRobustCommand\crfamily{\fontfamily{pcr}\selectfont}
\def\headingfont{\fontfamily{cmss}\selectfont}
\def\documentfancyfont{%
\gdef\headingfont{\crfamily}%
\fontfamily{ccr}\fontseries{m}\selectfont%
}
\def\documentfont{%
\gdef\headingfont{\fontfamily{cmss}\selectfont}%
\fontfamily{cmss}\fontseries{m}\selectfont%
\renewcommand{\sfdefault}{phv}%
\renewcommand{\ttdefault}{pcr}%
\renewcommand{\rmdefault}{cmr}% <— funktionieren nicht mit {ptm}
\renewcommand{\bfdefault}{bx}%
\renewcommand{\itdefault}{it}%
\renewcommand{\sldefault}{sl}%
\renewcommand{\scdefault}{sc}%
\renewcommand{\updefault}{n}%
}
\allowdisplaybreaks
\let\altcleardoublepage\cleardoublepage
\let\cleardoublepage\clearpage
\def\startdocumentlayoutoptions{
\selectlanguage{ngerman}
\setlength{\parskip}{0.5\baselineskip}
\setlength{\parindent}{0pt}
\kopfzeiledefault
\documentfont
\normalsize
}
\def\highlightTerm#1{\emph{#1}}
%% ********************************************************************************
%% FILE: src/setup-localmacros.tex
%% ********************************************************************************
%% ****************************************************************
%% MATHE:
%% ****************************************************************
\def\reell{\mathbb{R}}
\def\kmplx{\mathbb{C}}
\def\Torus{\mathbb{T}}
\def\rtnl{\mathbb{Q}}
\def\intgr{\mathbb{Z}}
\def\ntrl{\mathbb{N}}
\def\ntrlpos{\mathbb{N}}
\def\ntrlzero{\mathbb{N}_{0}}
\def\reellNonNeg{\reell_{+}}
\def\leer{\emptyset}
\def\restr#1{\vert_{#1}}
\def\ohne{\setminus}
\def\Pot{\mathop{\mathcal{P}}}
\def\brkt#1{\langle{}#1{}\rangle}
\def\lsim{\mathop{\sim}}
\def\lneg{\mathop{\neg}}
\def\land{\mathop{\wedge}}
\def\lor{\mathop{\vee}}
\def\eps{\varepsilon}
\let\altphi\phi
\let\altvarphi\varphi
\def\phi{\altvarphi}
\def\varphi{\altphi}
\def\span{\mathop{\text{\upshape Lin}}}
\def\dim{\mathop{\text{\upshape dim}}}
\def\onematrix{\text{\upshape\bfseries I}}
\def\zeromatrix{\text{\upshape\bfseries 0}}
\def\zerovector{\text{\upshape\bfseries 0}}
\def\graph{\mathop{\text{\textup Gph}}}
\def\id{\text{\textup id}}
\def\modfn{\mathop{\text{\textup mod}}}
\makeatother
\begin{document}
\startdocumentlayoutoptions
%% FRONTMATTER:
\thispagestyle{plain}
%% ********************************************************************************
%% FILE: front/index.tex
%% ********************************************************************************
%% ********************************************************************************
%% FILE: front/title.tex
%% ********************************************************************************
\begin{titlepage}
\null
\vraum
\noindent\rule{\linewidth}{2pt}
{\hraum\LARGE Lineare Algebra I\hraum}\\
{\hraum\LARGE $\oast$\,\rule[0.175\baselineskip]{0.65\linewidth}{1pt}\,$\oast$ \hraum}\\
{\hraum\Large Lösungen zu diversen Aufgaben im Kurs\hraum}
\noindent\rule{\linewidth}{2pt}
\vraum
\noindent
\hraum{\footnotesize Raj Dahya}\hraum\\
\hraum{\small \itshape Fakultät für Mathematik und Informatik/Institut für Philosophie}\hraum\\
\hraum{\small \itshape Universität Leipzig.}\hraum\\
\hraum{\small Wintersemester 2020/2021 }\hraum
\end{titlepage}
%% ********************************************************************************
%% FILE: front/foreword.tex
%% ********************************************************************************
\chapter*{Vorwort}
Dieses Dokument enthält Lösungsansätze zu den Übungsserien, Selbstkontrollenaufgaben, und Quizzes.
Diese werden natürlich \emph{nach} Abgabefristen hochgeladen
und dienen \emph{nicht} als Musterlösungen!
Der Zweck dieser Lösungen ist es vielmehr, Ansätze zu präsentieren,
mit denen man seine \emph{eigenen} Versuche vergleichen kann.
%% ********************************************************************************
%% FILE: front/contents.tex
%% ********************************************************************************
\kopfzeiledefault
\footnotesize
\setcounter{tocdepth}{1}
\def\contentsname{Inhaltsverzeichnis}
\tableofcontents
%% HAUPTTEXT:
%% ********************************************************************************
%% FILE: body/index.tex
%% ********************************************************************************
\setcounternach{part}{1}
\part{Übungsserien}
\def\chaptername{Übungsserie}
%% ********************************************************************************
%% FILE: body/uebung/ueb1.tex
%% ********************************************************************************
\setcounternach{chapter}{1}
\chapter[Woche 1]{Woche 1}
\label{ueb:1}
\textbf{ACHTUNG.}
Diese Lösungen dienen \emph{nicht} als Musterlösungen sondern eher als Referenz.
Hier wird eingehender gearbeitet, als generell verlangt wird.
Das Hauptziel hier ist, eine Variant anzubieten, gegen die man seine Versuche vergleichen kann.
%% AUFGABE 1-1
\let\altsectionname\sectionname
\def\sectionname{Aufgabe}
\section[Aufgabe 1]{}
\label{ueb:1:ex:1}
\let\sectionname\altsectionname
Zu bestimmen ist die Lösungsmenge
\begin{mathe}[mc]{rcl}
L_{\alpha,\beta} &:= &\{
\mathbf{x}\in\reell^{n}
\mid A_{\alpha}\mathbf{x}=\mathbf{b}_{\beta}
\}\\
\end{mathe}
für $\alpha,\beta\in\reell$,
wobei $m=3$ und $n=4$, und
$A_{\alpha}\in\reell^{m\times n}$ und $\mathbf{b}_{\beta}\in\reell^{m}$
durch
\begin{mathe}[mc]{rclqrcl}
A_{\alpha} &:= &\begin{matrix}{cccc}
1 &7 &2 &-1\\
1 &8 &6 &-3\\
2 &14 &\alpha &-2\\
\end{matrix}
&\mathbf{b}_{\beta} &:= &\begin{vector}4\\0\\\beta\\\end{vector}
\end{mathe}
gegeben sind.
Um die Lösungsmenge zu bestimmen führen wir das Gaußverfahren aus:
\begin{algorithm}[\rtab][\rtab]
Ursprüngliches LGS $(A_{\alpha}|b_{\beta})$:
\begin{mathe}[mc]{c}
\begin{matrix}{cccc|c}
1 &7 &2 &-1 &4\\
1 &8 &6 &-3 &0\\
2 &14 &\alpha &-2 &\beta\\
\end{matrix}\\
\end{mathe}
Wende die Zeilentransformationen
{\footnotesize
\begin{mathe}[mc]{rcl}
Z_{2} &\leftsquigarrow &Z_{2}-Z_{1}\\
Z_{3} &\leftsquigarrow &Z_{3}-2\cdot Z_{1}\\
\end{mathe}}
an:
\begin{mathe}[mc]{c}
\begin{matrix}{cccc|c}
\boxed{1} &7 &2 &-1 &4\\
0 &\boxed{1} &4 &-2 &-4\\
0 &0 &\boxed{\alpha - 4} &0 &\beta - 8\\
\end{matrix}\\
\end{mathe}
\end{algorithm}
Die eingezeichneten Einträge markieren die ersten Einträge der Stufen.
Es gibt also $2$ oder $3$ Stufen, je nachdem, ob ${\alpha - 4=0}$.
Dies führt zu einem Fallunterschied:
\begin{enumerate}{\bfseries {Fall} 1.}
%% FALL 1
\item $\alpha-4=0$. Das heißt, $\alpha=4$.
In diesem Falle hat das augmentierte System genau $2$ Stufen
und sieht wie folgt aus:
\begin{mathe}[mc]{c}
\begin{matrix}{cccc|c}
\boxed{1} &7 &2 &-1 &4\\
0 &\boxed{1} &4 &-2 &-4\\
0 &0 &0 &0 &\beta - 8\\
\end{matrix}\\
\end{mathe}
Dies führt zu zwei weiteren Fällen, denn die $3$. Gleichung ist jetzt genau dann lösbar,
wenn $\beta-8=0$.
\begin{enumerate}{\bfseries {Fall 1}a.}
%% FALL 1a
\item $\beta-8\neq 0$. Das heißt, $\beta\neq 8$.
Dann ist die $3$. Gleichung und damit das LGS nicht lösbar.
Darum erhalten wir $\boxed{L_{\alpha,\beta}=\leer}$.
%% FALL 1b
\item $\beta-8=0$. Das heißt, $\beta=8$.
Dann ist die $3$. Gleichung trivialerweise erfüllt.
Das augmentierte System sieht wird zum
\begin{mathe}[mc]{c}
\begin{matrix}{cccc|c}
\boxed{1} &7 &2 &-1 &4\\
0 &\boxed{1} &4 &-2 &-4\\
0 &0 &0 &0 &0\\
\end{matrix}\\
\end{mathe}
und kann jetzt aufgelöst werden.
Wir arbeiten von unten nach oben:
\begin{algorithm}[2\rtab][\rtab]
Aus der ganzen Zeilenstufenform erschließt sich
\begin{mathe}[mc]{c}
x_{3},\, x_{4}\,\text{sind frei}\\
\end{mathe}
Aus der Stufenform von Gleichungen $2$ und $1$ erschließt sich
\begin{mathe}[mc]{rcl}
x_{2} &= &-4 - 4x_{3} + 2x_{4}\\
x_{1} &= &4 - 7x_{2} - 2x_{3} + x_{4}\\
&= &4 - 7(-4 - 4x_{3} + 2x_{4}) - 2x_{3} + x_{4}\\
&= &32 + 26x_{3} + -13x_{4}\\
\end{mathe}
Zusammengefasst erhalten wir die allgemeine Form der Lösung:
\begin{mathe}[mc]{rcl}
\mathbf{x} &= &\begin{svector}x_{1}\\x_{2}\\x_{3}\\x_{4}\\\end{svector}\\
&= &\begin{svector}32 + 26x_{3} + -13x_{4}\\-4 - 4x_{3} + 2x_{4}\\x_{3}\\x_{4}\\\end{svector}\\
&= &\begin{svector}32 + 26x_{3} + -13x_{4}\\-4 - 4x_{3} + 2x_{4}\\0 + 1x_{3} + 0x_{4}\\0 + 0x_{3} + 1x_{4}\\\end{svector}\\
&= &\begin{svector}32\\-4\\0\\0\\\end{svector}
+ \begin{svector}26x_{3}\\-4x_{3}\\1x_{3}\\0x_{3}\\\end{svector}
+ \begin{svector}-13x_{4}\\2x_{4}\\1x_{4}\\1x_{4}\\\end{svector}\\
&= &\begin{svector}32\\-4\\0\\0\\\end{svector}
+ x_{3}\cdot\begin{svector}26\\-4\\1\\0\\\end{svector}
+ x_{4}\cdot\begin{svector}-13\\2\\1\\1\\\end{svector}\\
\end{mathe}
mit $x_{3}$, $x_{4}$ frei wählbar.
\end{algorithm}
Also erhalten wird in diesem Falle
$\boxed{
L_{\alpha,\beta}=\left\{
\begin{svector}32\\-4\\0\\0\\\end{svector}
+ t_{1}\cdot\begin{svector}26\\-4\\1\\0\\\end{svector}
+ t_{2}\cdot\begin{svector}-13\\2\\1\\1\\\end{svector}
\mid t_{1}, t_{2}\in\reell
\right\}
}$,
oder etwas kompakter formuliert,
${L_{\alpha,\beta}=\begin{svector}32\\-4\\0\\0\\\end{svector} + \span\left\{\begin{svector}26\\-4\\1\\0\\\end{svector}, \begin{svector}-13\\2\\1\\1\\\end{svector}\right\}}$.
\end{enumerate}
%% FALL 2
\item $\alpha-4\neq 0$. Das heißt, $\alpha\neq 4$.
In diesem Falle hat das augmentierte System genau $3$ Stufen und diesmal ist nur $x_{4}$ frei.
Man beachte, dass dies im Grunde genau wie Fall 1b ist, nur dass wir zusätzlich Gleichung 3 beachten und $x_{3}$ bestimmen müssen.
\begin{algorithm}[2\rtab][\rtab]
Aus der Stufenform von Gleichungen $3$ ergibt sich
\begin{mathe}[mc]{rcl}
x_{3} &= &\frac{\beta-8}{\alpha-4}\\
\end{mathe}
Der Rest der Lösung des Gleichungssystems verhält sich genau wie im Fall 3b,
das heißt
\begin{mathe}[mc]{rcl}
\mathbf{x} &= &\begin{svector}32\\-4\\0\\0\\\end{svector}
+ x_{3}\cdot\begin{svector}26\\-4\\1\\0\\\end{svector}
+ x_{4}\cdot\begin{svector}-13\\2\\1\\1\\\end{svector}\\
&= &\begin{svector}32\\-4\\0\\0\\\end{svector}
+ \frac{\beta-8}{\alpha-4}\cdot\begin{svector}26\\-4\\1\\0\\\end{svector}
+ x_{4}\cdot\begin{svector}-13\\2\\1\\1\\\end{svector},\\
\end{mathe}
wobei $x_{4}$ frei wählbar ist.
\end{algorithm}
Also erhalten wird in diesem Falle
$\boxed{
L_{\alpha,\beta}=\left\{
\begin{svector}32\\-4\\0\\0\\\end{svector}
+ \frac{\beta-8}{\alpha-4}\cdot\begin{svector}26\\-4\\1\\0\\\end{svector}
+ t\cdot\begin{svector}-13\\2\\1\\1\\\end{svector}
\mid t\in\reell
\right\}
}$,
oder etwas kompakter formuliert,
${L_{\alpha,\beta}=\begin{svector}32\\-4\\0\\0\\\end{svector} + \frac{\beta-8}{\alpha-4}\cdot\begin{svector}26\\-4\\1\\0\\\end{svector} + \span\left\{\begin{svector}-13\\2\\1\\1\\\end{svector}\right\}}$.
\end{enumerate}
Wir fassen die Lösung für alle Fälle zusammen:
\begin{mathe}[mc]{rcl}
L_{\alpha,\beta} &= &\begin{cases}[m]{lcl}
\leer &: &\alpha=4,\,\beta\neq 8\\
\mathbf{u} + \span\{\mathbf{v},\mathbf{w}\} &: &\alpha=4,\,\beta=8\\
\mathbf{u} + \frac{\alpha-4}{\beta-8}\mathbf{v} + \span\{\mathbf{w}\} &: &\alpha\neq 4\\
\end{cases}
\end{mathe}
für alle $\alpha,\beta\in\reell$,
wobei
$\mathbf{u} = \begin{svector}32\\-4\\0\\0\\\end{svector}$,
$\mathbf{v} = \begin{svector}26\\-4\\1\\0\\\end{svector}$,
$\mathbf{w} = \begin{svector}-13\\2\\1\\1\\\end{svector}$.
%% AUFGABE 1-2
\let\altsectionname\sectionname
\def\sectionname{Aufgabe}
\section[Aufgabe 2]{}
\label{ueb:1:ex:2}
\let\sectionname\altsectionname
\begin{schattierteboxdunn}
\begin{satz}
\makelabel{satz:main:ueb:1:ex:2}
Angewandt auf die erweiterte Koeffizientenmatrix eines linearen Gleichungssystems
verändern
die elementaren Zeilenumformungen vom Typ (I), (II) und (III)
die Menge der Lösungen nicht.
\end{satz}
\end{schattierteboxdunn}
Wir beweisen \Cref{satz:main:ueb:1:ex:2} mithilfe der folgenden Teilergebnisse.
\begin{lemm}
\makelabel{lemm:1:ueb:1:ex:2}
Seien $m,n\in\ntrlpos$ und $A\in\reell^{m\times n}$ und $\mathbf{b}\in\reell^{m}$.
Für $i,j\in\{1,2,\ldots,m\}$ mit $i\neq j$ bezeichne mit
\begin{mathe}[mc]{rcl}
(A|\mathbf{b}) &\overset{I;i,j}{\rightsquigarrow} &(A'|\mathbf{b}')\\
\end{mathe}
die Anwendung von Zeilentransformation (I) auf $(A|\mathbf{b})$,
wobei Zeile${}_{i}$ und Zeile${}_{j}$ umgetauscht werden,
was in $(A'|\mathbf{b}')$ resultiert.
Dann für alle ${\mathbf{x}\in\reell^{n}}$,
falls $\mathbf{x}$ eine Lösung für $(A|\mathbf{b})$ ist,
dann ist $\mathbf{x}$ eine Lösung für $(A'|\mathbf{b}')$.
\end{lemm}
\begin{einzug}[\rtab][\rtab]
\begin{proof}
Betrachte den Fall $i<j$.
Es gilt
\begin{longtable}[mc]{RL}
&\text{$\mathbf{x}$ eine Lösung für $(A|\mathbf{b})$}\\
\Longrightarrow
&{\scriptsize
\left\{
\begin{array}[m]{crccccclcl}
&(a_{1,1}x_{1} &+ &a_{1,2}x_{2} &+ &\cdots &+ &a_{1,n}x_{n} &= &b_{1})\\
\text{und} &(a_{2,1}x_{1} &+ &a_{2,2}x_{2} &+ &\cdots &+ &a_{2,n}x_{n} &= &b_{2})\\
\cdots\\
\text{und} &(a_{i,1}x_{1} &+ &a_{i,2}x_{2} &+ &\cdots &+ &a_{i,n}x_{n} &= &b_{i})\\
\cdots\\
\text{und} &(a_{j,1}x_{1} &+ &a_{j,2}x_{2} &+ &\cdots &+ &a_{j,n}x_{n} &= &b_{j})\\
\cdots\\
\text{und} &(a_{m,1}x_{1} &+ &a_{m,2}x_{2} &+ &\cdots &+ &a_{m,n}x_{n} &= &b_{m})
\end{array}
\right.}\\
\\
\Longrightarrow
&{\scriptsize
\left\{
\begin{array}[m]{crccccclcl}
&(a_{1,1}x_{1} &+ &a_{1,2}x_{2} &+ &\cdots &+ &a_{1,n}x_{n} &= &b_{1})\\
\text{und} &(a_{2,1}x_{1} &+ &a_{2,2}x_{2} &+ &\cdots &+ &a_{2,n}x_{n} &= &b_{2})\\
\cdots\\
\text{und} &(a_{j,1}x_{1} &+ &a_{j,2}x_{2} &+ &\cdots &+ &a_{j,n}x_{n} &= &b_{j})\\
\cdots\\
\text{und} &(a_{i,1}x_{1} &+ &a_{i,2}x_{2} &+ &\cdots &+ &a_{i,n}x_{n} &= &b_{i})\\
\cdots\\
\text{und} &(a_{m,1}x_{1} &+ &a_{m,2}x_{2} &+ &\cdots &+ &a_{m,n}x_{n} &= &b_{m})
\end{array}
\right.}\\
\\
&\text{da lediglich zwei Aussagen in einer Konjunktion umgetauscht werden}\\
\\
\Longrightarrow
&\text{$\mathbf{x}$ eine Lösung für $(A'|\mathbf{b})'$, da $(A|\mathbf{b})\overset{I;i,j}{\rightsquigarrow}(A'|\mathbf{b}')$.}\\
\end{longtable}
Der Fall $i>j$ lässt sich analog zeigen.
Falls $i=j$ bleibt das System unverändert, sodass die Behauptung trivialerweise gilt.
\end{proof}
\end{einzug}
\begin{lemm}
\makelabel{lemm:2:ueb:1:ex:2}
Seien $m,n\in\ntrlpos$ und $A\in\reell^{m\times n}$ und $\mathbf{b}\in\reell^{m}$.
Für ${i\in\{1,2,\ldots,m\}}$ und ${\alpha\in\reell\ohne\{0\}}$ bezeichne mit
\begin{mathe}[mc]{rcl}
(A|\mathbf{b}) &\overset{II;i,\alpha}{\rightsquigarrow} &(A'|\mathbf{b}')\\
\end{mathe}
die Anwendung von Zeilentransformation (II) auf $(A|\mathbf{b})$,
wobei Zeile${}_{i}$ durch $\alpha\cdot$Zeile${}_{i}$ ersetzt wird,
was in $(A'|\mathbf{b}')$ resultiert.
Dann für alle ${\mathbf{x}\in\reell^{n}}$,
falls $\mathbf{x}$ eine Lösung für $(A|\mathbf{b})$ ist,
dann ist $\mathbf{x}$ eine Lösung für $(A'|\mathbf{b}')$.
\end{lemm}
\begin{einzug}[\rtab][\rtab]
\begin{proof}
Es gilt
\begin{longtable}[mc]{RL}
&\text{$\mathbf{x}$ eine Lösung für $(A|\mathbf{b})$}\\
\Longrightarrow
&{\scriptsize
\left\{
\begin{array}[m]{crccccclcl}
&(a_{1,1}x_{1} &+ &a_{1,2}x_{2} &+ &\cdots &+ &a_{1,n}x_{n} &= &b_{1})\\
\text{und} &(a_{2,1}x_{1} &+ &a_{2,2}x_{2} &+ &\cdots &+ &a_{2,n}x_{n} &= &b_{2})\\
\cdots\\
\text{und} &(a_{i,1}x_{1} &+ &a_{i,2}x_{2} &+ &\cdots &+ &a_{i,n}x_{n} &= &b_{i})\\
\cdots\\
\text{und} &(a_{m,1}x_{1} &+ &a_{m,2}x_{2} &+ &\cdots &+ &a_{m,n}x_{n} &= &b_{m})
\end{array}
\right.}\\
\\
\Longrightarrow
&{\scriptsize
\left\{
\begin{array}[m]{crccccclcl}
&(a_{1,1}x_{1} &+ &a_{1,2}x_{2} &+ &\cdots &+ &a_{1,n}x_{n} &= &b_{1})\\
\text{und} &(a_{2,1}x_{1} &+ &a_{2,2}x_{2} &+ &\cdots &+ &a_{2,n}x_{n} &= &b_{2})\\
\cdots\\
\text{und} &(\alpha\cdot (a_{i,1}x_{1} &+ &a_{i,2}x_{2} &+ &\cdots &+ &a_{i,n}x_{n}) &= &\alpha\cdot b_{i})\\
\cdots\\
\text{und} &(a_{m,1}x_{1} &+ &a_{m,2}x_{2} &+ &\cdots &+ &a_{m,n}x_{n} &= &b_{m})
\end{array}
\right.}\\
\\
\Longrightarrow
&{\scriptsize
\left\{
\begin{array}[m]{crccccclcl}
&(a_{1,1}x_{1} &+ &a_{1,2}x_{2} &+ &\cdots &+ &a_{1,n}x_{n} &= &b_{1})\\
\text{und} &(a_{2,1}x_{1} &+ &a_{2,2}x_{2} &+ &\cdots &+ &a_{2,n}x_{n} &= &b_{2})\\
\cdots\\
\text{und} &(\alpha\cdot a_{i,1}x_{1} &+ &\alpha\cdot a_{i,2}x_{2} &+ &\cdots &+ &\alpha\cdot a_{i,n}x_{n} &= &\alpha\cdot b_{i})\\
\cdots\\
\text{und} &(a_{m,1}x_{1} &+ &a_{m,2}x_{2} &+ &\cdots &+ &a_{m,n}x_{n} &= &b_{m})
\end{array}
\right.}\\
\\
&\text{$\mathbf{x}$ eine Lösung für $(A'|\mathbf{b})'$, da $(A|\mathbf{b})\overset{II;i,\alpha}{\rightsquigarrow}(A'|\mathbf{b}')$.}
\end{longtable}
Also gilt die Behauptung.
\end{proof}
\end{einzug}
\begin{lemm}
\makelabel{lemm:3:ueb:1:ex:2}
Seien $m,n\in\ntrlpos$ und $A\in\reell^{m\times n}$ und $\mathbf{b}\in\reell^{m}$.
Für ${i,j\in\{1,2,\ldots,m\}}$ mit $i\neq j$ und $\alpha\in\reell$ bezeichne mit
\begin{mathe}[mc]{rcl}
(A|\mathbf{b}) &\overset{III;i,j,\alpha}{\rightsquigarrow} &(A'|\mathbf{b}')\\
\end{mathe}
die Anwendung von Zeilentransformation (III) auf $(A|\mathbf{b})$,
wobei Zeile${}_{i}$ durch die Addition von Zeile${}_{i}$ mit $\alpha\cdot$Zeile${}_{j}$ ersetzt wird,
was in $(A'|\mathbf{b}')$ resultiert.
Dann für alle ${\mathbf{x}\in\reell^{n}}$,
falls $\mathbf{x}$ eine Lösung für $(A|\mathbf{b})$ ist,
dann ist $\mathbf{x}$ eine Lösung für $(A'|\mathbf{b}')$.
\end{lemm}
\begin{einzug}[\rtab][\rtab]
\begin{proof}
Es gilt
\begin{longtable}[mc]{RL}
&\text{$\mathbf{x}$ eine Lösung für $(A|\mathbf{b})$}\\
\Longrightarrow
&{\scriptsize
\left\{
\begin{array}[m]{crccccclcl}
&(a_{1,1}x_{1} &+ &a_{1,2}x_{2} &+ &\cdots &+ &a_{1,n}x_{n} &= &b_{1})\\
\text{und} &(a_{2,1}x_{1} &+ &a_{2,2}x_{2} &+ &\cdots &+ &a_{2,n}x_{n} &= &b_{2})\\
\cdots\\
\text{und} &(a_{i,1}x_{1} &+ &a_{i,2}x_{2} &+ &\cdots &+ &a_{i,n}x_{n} &= &b_{i})\\
\cdots\\
\text{und} &(a_{m,1}x_{1} &+ &a_{m,2}x_{2} &+ &\cdots &+ &a_{m,n}x_{n} &= &b_{m})
\end{array}
\right.}\\
\\
\Longrightarrow
&{\scriptsize
\left\{
\begin{array}[m]{crccccclcl}
&(a_{1,1}x_{1} &+ &a_{1,2}x_{2} &+ &\cdots &+ &a_{1,n}x_{n} &= &b_{1})\\
\text{und} &(a_{2,1}x_{1} &+ &a_{2,2}x_{2} &+ &\cdots &+ &a_{2,n}x_{n} &= &b_{2})\\
\cdots\\
\text{und} &(a_{i,1}x_{1} &+ &a_{i,2}x_{2} &+ &\cdots &+ &a_{i,n}x_{n} + \alpha\cdot b_{j} &= &b_{i} + \alpha\cdot b_{j})\\
\cdots\\
\text{und} &(a_{m,1}x_{1} &+ &a_{m,2}x_{2} &+ &\cdots &+ &a_{m,n}x_{n} &= &b_{m})
\end{array}
\right.}\\
\\
\Longrightarrow
&{\scriptsize
\left\{
\begin{array}[m]{crccccclcl}
&(a_{1,1}x_{1} &+ &a_{1,2}x_{2} &+ &\cdots &+ &a_{1,n}x_{n} &= &b_{1})\\
\text{und} &(a_{2,1}x_{1} &+ &a_{2,2}x_{2} &+ &\cdots &+ &a_{2,n}x_{n} &= &b_{2})\\
\cdots\\
\text{und} &(a_{i,1}x_{1} &+ &a_{i,2}x_{2} &+ &\cdots &+ &a_{i,n}x_{n}\\
&+\alpha\cdot a_{j,1}x_{1} &+ &\alpha\cdot a_{j,2}x_{2} &+ &\cdots &+ &\alpha\cdot a_{j,n}x_{n} &= &b_{i} + \alpha\cdot b_{j})\\
\cdots\\
\text{und} &(a_{m,1}x_{1} &+ &a_{m,2}x_{2} &+ &\cdots &+ &a_{m,n}x_{n} &= &b_{m})
\end{array}
\right.}\\
\\
&\text{da laut der $j$-ten Gleichung gilt ${b_{j}=\sum_{k=1}^{m}a_{j,k}x_{k}}$}\\
\\
\Longrightarrow
&{\scriptsize
\left\{
\begin{array}[m]{crccccclcl}
&(a_{1,1}x_{1} &+ &a_{1,2}x_{2} &+ &\cdots &+ &a_{1,n}x_{n} &= &b_{1})\\
\text{und} &(a_{2,1}x_{1} &+ &a_{2,2}x_{2} &+ &\cdots &+ &a_{2,n}x_{n} &= &b_{2})\\
\cdots\\
\text{und} &(a'_{i,1}x_{1} &+ &a'_{i,2}x_{2} &+ &\cdots &+ &a'_{i,n}x_{n} &= &b'_{i})\\
\cdots\\
\text{und} &(a_{m,1}x_{1} &+ &a_{m,2}x_{2} &+ &\cdots &+ &a_{m,n}x_{n} &= &b_{m}),
\end{array}
\right.}\\
\\
&\text{wobei $a'_{i,k}=a_{i,k}+\alpha\cdot a_{j,k}$ für alle $k$ und $b'_{i}=b_{i}+\alpha\cdot b_{j}$}\\
\\
\Longrightarrow
&\text{$\mathbf{x}$ eine Lösung für $(A'|\mathbf{b})'$, da $(A|\mathbf{b})\overset{III;i,j,\alpha}{\rightsquigarrow}(A'|\mathbf{b}')$.}
\end{longtable}
Also gilt die Behauptung.
\end{proof}
\end{einzug}
Endlich können wir \Cref{satz:main:ueb:1:ex:2} beweisen:
\begin{proof}[von \Cref{satz:main:ueb:1:ex:2}]
Seien $m,n\in\ntrlpos$ und $A\in\reell^{m\times n}$ und $\mathbf{b}\in\reell^{m}$.
Seien $A'\in\reell^{m\times n}$ und $\mathbf{b}'\in\reell^{m}$,
so dass $(A|\mathbf{b})$ durch eine Transformation der Art (I), (II) oder (III)
aus $(A|\mathbf{b})$ entsteht.
Das heißt, entweder
\begin{mathe}[mc]{lrcl}
\eqtag[eq:0:\beweislabel]
&(A|\mathbf{b}) &\overset{I;i,j}{\rightsquigarrow} &(A'|\mathbf{b}')\\
\text{oder} &(A|\mathbf{b}) &\overset{I;i,\alpha}{\rightsquigarrow} &(A'|\mathbf{b}')\\
\text{oder} &(A|\mathbf{b}) &\overset{III;i,j,\alpha}{\rightsquigarrow} &(A'|\mathbf{b}')\\
\end{mathe}
gilt, für ein $i,j\in\{1,2,\ldots,m\}$ mit $i\neq j$ und $\alpha\in\reell\ohne\{0\}$.\\
\textbf{Zu zeigen:}
\begin{mathe}[mc]{rcl}
\eqtag[eq:1:\beweislabel]
\{\mathbf{x}\in\reell^{n}\mid\mathbf{x}\text{ eine Lösung für }(A|\mathbf{b})\}
&= &\{\mathbf{x}\in\reell^{n}\mid\mathbf{x}\text{ eine Lösung für }(A|\mathbf{b})\}.\\
\end{mathe}
Wir zeigen dies in zwei Teile:
\uline{\bfseries ($\subseteq$.)}\\
Sei $\mathbf{x}\in\reell^{n}$ ein beliebiges Element aus der linken Menge,
d.\,h. $\mathbf{x}$ ist eine Lösung zu $(A|\mathbf{b})$.
Laut \Cref{lemm:1:ueb:1:ex:2} + \Cref{lemm:2:ueb:1:ex:2} + \Cref{lemm:3:ueb:1:ex:2}
und wegen \eqcref{eq:0:\beweislabel}
erhalten wir, dass $\mathbf{x}$ eine Lösung zu $(A'|\mathbf{b}')$ ist,
d.\,h. $\mathbf{x}$ liegt in der rechten Menge.
Also ist die linke Menge in der rechten enthalten.
\uline{\bfseries ($\supseteq$.)}\\
Man beachte zuerst, dass sich die Transformation in \eqcref{eq:0:\beweislabel} umkehren lässt---\text{und zwar durch Elementartransformationen}.
Es ist einfach zu sehen, dass entweder
\begin{mathe}[mc]{lrcl}
&(A'|\mathbf{b}') &\overset{I;i,j}{\rightsquigarrow} &(A|\mathbf{b})\\
\text{oder} &(A'|\mathbf{b}') &\overset{I;i,\alpha^{-1}}{\rightsquigarrow} &(A|\mathbf{b})\\
\text{oder} &(A'|\mathbf{b}') &\overset{III;i,j,-\alpha}{\rightsquigarrow} &(A|\mathbf{b}).\\
\end{mathe}
Die Situation ist also analog zum $\subseteq$-Teil.
Darum gilt die $\supseteq$-Inklusion in \eqcref{eq:1:\beweislabel}.
\end{proof}
\clearpage
%% AUFGABE 1-3
\let\altsectionname\sectionname
\def\sectionname{Aufgabe}
\section[Aufgabe 3]{}
\label{ueb:1:ex:3}
\let\sectionname\altsectionname
Für diese Aufgabe wird das Konzept der \emph{linearen Unabhängigkeit} aus Kapitel 5 angewandt.
\begin{defn}
Seien $m,n\in\ntrlpos$ mit $m>n$
und seien $A\in\reell^{m\times n}$, $\mathbf{b}\in\reell^{m}$,
und $I\subseteq\{1,2,\ldots,m\}$.
Bezeichne mit $(A|\mathbf{b})_{I}$ die erweiterte Koeffizientenmatrix $(A|\mathbf{b})$,
die auf die Zeilen mit Indexes aus $I$ (in bspw. aufsteigender Reihenfolge) reduziert ist.
\end{defn}
\begin{e.g.}
Für $(A|\mathbf{b})$ gleich
{\scriptsize
\begin{mathe}[mc]{c}
\begin{matrix}{ccc|c}
-5 &0 &0 &-7\\
4 &-6 &-10 &6\\
-2 &-6 &-6 &9\\
-7 &4 &-1 &-5\\
4 &-5 &2 &-9\\
-5 &8 &-7 &-5\\
\end{matrix}
\end{mathe}}
und $I=\{2,5,6\}$ ist $(A|\mathbf{b})_{I}$ gleich
{\scriptsize
\begin{mathe}[bc]{c}
\begin{matrix}{ccc|c}
4 &-6 &-10 &6\\
4 &-5 &2 &-9\\
-5 &8 &-7 &-5\\
\end{matrix}.
\end{mathe}}
\nvraum{1}
\end{e.g.}
Mit diesem Mittel können wir nun die Hauptaussage in der Aufgabe formulieren:
\begin{schattierteboxdunn}
\begin{satz}
\makelabel{satz:main:ueb:1:ex:3}
Seien $m,n\in\ntrlpos$ mit $m>n$
und seien $A\in\reell^{m\times n}$ und $\mathbf{b}\in\reell^{m}$.
Falls $(A|\mathbf{b})$ unlösbar ist,
dann existiert $I\subseteq\{1,2,\ldots,m\}$ mit $|I|=n+1$,
so dass $(A|\mathbf{b})_{I}$ unlösbar ist.
\end{satz}
\end{schattierteboxdunn}
\begin{einzug}[\rtab][\rtab]
\begin{proof}[*][\Cref{\beweislabel}]
Es stehen nun die \emph{Zeilen} der Matrix $A$ im Fokus.
Wir verwandeln diese in Vektoren, d.\,h. setze
\begin{mathe}[mc]{c}
\mathbf{z}^{(i)}\in\reell^{n}\,\text{die $i$-te Zeile von $A$ als Vektor geschrieben}
\end{mathe}
für $i\in\{1,2,\ldots,m\}$.
Da ${\mathbf{z}^{(1)},\mathbf{z}^{(2)},\ldots,\mathbf{z}^{(m)}\in\reell^{n}}$,
können wir eine \emph{maximale Menge} ${I_{0}\subseteq\{1,2,\ldots,m\}}$ finden,
so dass $(\mathbf{z}^{(i)})_{i\in I_{0}}$ linear unabhängige Vektoren sind.
Aus der Maximalität folgt, dass für jedes ${k\in\{1,2,\ldots,m\}\ohne I_{0}}$
$(\mathbf{z}^{(i)})_{i\in I_{0}\cup\{k\}}$ \emph{linear abhängig} sind.
Wegen der Dimension von $\reell^{n}$ gilt ${|I|\leq\min\{m,n\}=n}$.
Aus der linearer Unabhängigkeit von den $(\mathbf{z}^{(i)})_{i\in I_{0}}$ folgt,
dass es (eindeutige) Koeffizienten $c_{k,i}\in\reell$ für $i\in I_{0}$ gibt,
so dass
\begin{mathe}[mc]{rcl}
\eqtag[eq:1:\beweislabel]
\mathbf{z}^{(k)} &= &\sum_{i\in I_{0}:~}c_{k,i}\mathbf{z}^{(i)}\\
\end{mathe}
gilt.
Um nun die Hauptaussage zu zeigen, nehmen wir an, dass $(A|\mathbf{b})$ unlösbar ist.
\textbf{Zu zeigen:} Es gibt eine Teilmenge ${I\subseteq\{1,2,\ldots,m\}}$ mit ${|I|=n+1}$,
so dass $(A|\mathbf{b})_{I}$ unlösbar ist.
\fbox{Angenommen, dies sei nicht der Fall.}
Aus dieser Annahme leiten wir folgende Behauptungen ab:
\begin{kompaktitem}[\rtab][\rtab]
\behauptungbeleg{1}
Die Verhältnisse zwischen den Zeilenvektoren in \eqcref{eq:1:\beweislabel} gelten auch für die Einträge aus $\mathbf{b}$.
Das heißt
\begin{mathe}[mc]{rcl}
\eqtag[eq:2:\beweislabel]
b_{k} &= &\sum_{i\in I_{0}:~}c_{k,i}b_{i}\\
\end{mathe}
für alle ${k\in\{1,2,\ldots,m+1\}\ohne I_{0}}$.\\
\voritemise
\belegbehauptung
Sei $k\in\{1,2,\ldots,m+1\}\ohne I_{0}$ beliebig.
Da $|I_{0}|\leq n<n+1$ lässt sich eine Teilmenge $I\subseteq\{1,2,\ldots,m\}$ wählen,
mit $I\supseteq I_{0}\cup\{k\}$ und $|I|=n+1$.
Dann per \emph{Annahme} ist $(A|\mathbf{b})_{I}$ lösbar.
Das heißt, $\mathbf{x}\in\reell^{n}$ existiert, so dass
\begin{mathe}[mc]{rcl}
\eqtag[eq:3:\beweislabel]
b_{i} &= &\sum_{j=1}^{n}a_{i,j}x_{j}\\
\end{mathe}
für alle $i\in I$ gilt.
Da $k\in I$ und $I_{0}\subseteq I$ und wegen \eqcref{eq:1:\beweislabel} erhalten wir
nun das Verhältnis
\begin{longmathe}[mc]{RCL}
b_{k} &= &\sum_{j=1}^{n}a_{k,j}x_{j}\\
&= &\sum_{j=1}^{n}(\mathbf{z}^{(k)})_{j}x_{j}\\
&&\quad\text{da die Einträge der $k$-ten Zeile den Einträgen von $\mathbf{z}^{(k)}$ entsprechen}\\
&\eqcrefoverset{eq:1:\beweislabel}{=}
&\sum_{j=1}^{n}(\sum_{i\in I_{0}}c_{k,i}\mathbf{z}^{(i)})_{j}x_{j}\\
&= &\sum_{j=1}^{n}\sum_{i\in I_{0}}c_{k,i}z^{(i)}_{j}x_{j}\\
&= &\sum_{i\in I_{0}}c_{k,i}\sum_{j=1}^{n}z^{(i)}_{j}x_{j}\\
&= &\sum_{i\in I_{0}}c_{k,i}\sum_{j=1}^{n}a_{i,j}x_{j}\\
&&\quad\text{da die Einträge der $i$-ten Zeile den Einträgen von $\mathbf{z}^{(i)}$ entsprechen}\\
&\eqcrefoverset{eq:3:\beweislabel}{=} &\sum_{i\in I_{0}}c_{k,i}b_{i}.\\
\end{longmathe}
Darum gilt die Behauptung.
\enndeOfSomething[Beh. 1]
\behauptungbeleg{2}
Es gibt eine Lösung zu $(A|\mathbf{b})$.\\
\voritemise
\belegbehauptung
Da $|I_{0}|\leq n<n+1$ lässt sich eine Teilmenge $I\subseteq\{1,2,\ldots,m\}$ wählen,
so dass $I\supseteq I_{0}$ und $|I|=n+1$.
Dann per \emph{Annahme} ist $(A|\mathbf{b})_{I}$ lösbar.
Das heißt, ein $\mathbf{x}\in\reell^{n}$ existiert, so dass
\begin{mathe}[mc]{rcl}
\eqtag[eq:3b:\beweislabel]
b_{i} &= &\sum_{j=1}^{n}a_{i,j}x_{j}\\
\end{mathe}
für alle $i\in I$ gilt.
Da $I\supseteq I_{0}$ können wir \textbf{Behauptung 1} und die Verhältnisse in \eqcref{eq:1:\beweislabel} anwenden.
Für jedes ${k\in\{1,2,\ldots,m\}\ohne I}$ gilt
\begin{longmathe}[mc]{RCL}
\sum_{j=1}^{n}a_{k,j}x_{j}
&= &\sum_{j=1}^{n}(\mathbf{z}^{(k)})_{j}x_{j}\\
&&\quad\text{da die Einträge der $k$-ten Zeile den Einträgen von $\mathbf{z}^{(k)}$ entsprechen}\\
&\eqcrefoverset{eq:1:\beweislabel}{=}
&\sum_{j=1}^{n}(\sum_{i\in I_{0}}c_{k,i}\mathbf{z}^{(i)})_{j}x_{j}\\
&= &\sum_{j=1}^{n}\sum_{i\in I_{0}}c_{k,i}z^{(i)}_{j}x_{j}\\
&= &\sum_{i\in I_{0}}c_{k,i}\sum_{j=1}^{n}z^{(i)}_{j}x_{j}\\
&= &\sum_{i\in I_{0}}c_{k,i}\sum_{j=1}^{n}a_{i,j}x_{j}\\
&&\quad\text{da die Einträge der $i$-ten Zeile den Einträgen von $\mathbf{z}^{(i)}$ entsprechen}\\
&\eqcrefoverset{eq:3b:\beweislabel}{=} &\sum_{i\in I_{0}}c_{k,i}b_{i}\\
&\textoverset{Beh. 1}{=} &b_{k}\\
\end{longmathe}
Also ist $\mathbf{x}\in\reell^{n}$ nicht nur eine Lösung zu Zeile $i$ des LGS, $(A|\mathbf{b})$, für jedes $i\in I$,
sondern auch für jedes ${i\in\{1,2,\ldots,m\}\ohne I}$.
Das heißt, $\mathbf{x}$ ist eine Lösung des LGS $(A|\mathbf{b})$.
Also ist $(A|\mathbf{b})$ lösbar.
\enndeOfSomething[Beh. 2]
\end{kompaktitem}
Laut \textbf{Behauptung 2} ist also $(A|\mathbf{b})$ lösbar.
Dies ist aber ein Widerspruch!
Darum stimmt die \emph{Annahme} oben nicht.
Also gibt es \emph{doch} eine Teilmenge ${I\subseteq\{1,2,\ldots,m\}}$ mit ${|I|=n+1}$, so dass $(A|\mathbf{b})_{I}$ unlösbar ist.
Damit wurde die zu zeigende Implikation bewiesen.
\end{proof}
\end{einzug}
\begin{rem}
Falls man sich aber auf rudimentäre Mitteln beschränken will, kann man alternativ wie folgt vorgehen.
Man wende zuerst das Gaußverfahren an und erhalte somit eine Folge
\begin{mathe}[mc]{rcccccccl}
(A^{(0)}|\mathbf{b}^{(0)})
&\rightsquigarrow
&(A^{(1)}|\mathbf{b}^{(1)})
&\rightsquigarrow
&(A^{(2)}|\mathbf{b}^{(2)})
&\rightsquigarrow
&\cdots
&\rightsquigarrow
&(A^{(N)}|\mathbf{b}^{(N)})
\end{mathe}
wobei $N\in\ntrl$, ${A^{(0)}=A}$, ${\mathbf{b}^{(0)}=\mathbf{b}}$,
$(A^{(N)}|\mathbf{b}^{(N)})$ eine erweiterte Koeffizientenmatrix in Zeilenstufenform ist,
und jede der »$\rightsquigarrow$« Übergänge jeweils eine Transformation der Art (I), (II), oder (III) bezeichnet.
Da $m>n$ sieht nun die Zeilenstufenform, also $(A^{(N)}|\mathbf{b}^{(N)})$, folgendermaßen aus:
{\scriptsize
\begin{mathe}[mc]{rcl}
\begin{matrix}{cccccccc|c}
\underbrace{0\,0\,\ldots\,0}_{\ell_{1}} &\gamma_{1} &\cdots\cdots &\ast &\cdots\cdots &\cdots\cdots &\ast &\cdots\cdots &b^{(N)}_{1}\\
0\,0\,\ldots\,0 &0 &\underbrace{0\,0\,\ldots\,0}_{\ell_{2}} &\gamma_{2} &\cdots\cdots &\cdots\cdots &\ast &\cdots\cdots &b^{(N)}_{2}\\
\vdots & & & & & & &\vdots\\
0\,0\,\ldots\,0 &0 &0\,0\,\ldots\,0 &0 &\cdots\cdots &\underbrace{0\,0\,\ldots\,0}_{\ell_{r}} &\gamma_{r} &\cdots\cdots &b^{(N)}_{r}\\
0\,0\,\ldots\,0 &0 &0\,0\,\ldots\,0 &0 &\cdots\cdots &0\,0\,\ldots\,0 &0 &\cdots\cdots &b^{(N)}_{r+1}\\
\vdots & & & & & & &\vdots\\
0\,0\,\ldots\,0 &0 &0\,0\,\ldots\,0 &0 &\cdots\cdots &0\,0\,\ldots\,0 &0 &\cdots\cdots &b^{(N)}_{m}\\
\end{matrix}
\end{mathe}}
wobei $r\in\ntrlzero$ die Anzahl der Stufen ist,
${\ell_{1},\ell_{2},\ldots,\ell_{r}\in\ntrlzero}$,
und $\gamma_{1},\gamma_{2},\ldots,\gamma_{r}\in\reell\ohne\{0\}$ die Hauptkoeffizienten der Stufen sind.
Es muss nun $0\leq r\leq \min\{m,n\}=n$ gelten.
Jetzt kann man leicht dafür argumentiere, dass (1) die Zeilenstufenform, $(A^{(N)}|\mathbf{b}^{(N)})$, die Implikation erfüllt.
Dann aufgrund der Umkehrbarkeit der Elementartransformationen, reicht es aus zu zeigen, dass (2):
wenn ${(A',\mathbf{b}')\rightsquigarrow(A'',\mathbf{b}'')}$ und wenn $(A',\mathbf{b}')$ die Implikation erfüllt,
dann erfüllt $(A'',\mathbf{b}'')$ die Implikation.
Dies ist nur etwas mühseliger und die Argumentation von (2) führt letzten Endes zu ähnlichen Ideen, die im Beweis oben vorkommen.
\end{rem}
%% ********************************************************************************
%% FILE: body/uebung/ueb2.tex
%% ********************************************************************************
\setcounternach{chapter}{2}
\chapter[Woche 2]{Woche 2}
\label{ueb:2}
\textbf{ACHTUNG.}
Diese Lösungen dienen \emph{nicht} als Musterlösungen sondern eher als Referenz.
Hier wird eingehender gearbeitet, als generell verlangt wird.
Das Hauptziel hier ist, eine Variant anzubieten, gegen die man seine Versuche vergleichen kann.
%% AUFGABE 2-1
\let\altsectionname\sectionname
\def\sectionname{Aufgabe}
\section[Aufgabe 1]{}
\label{ueb:2:ex:1}
\let\sectionname\altsectionname
\begin{schattierteboxdunn}
\begin{satz}[vgl. {\cite[Korollar 1.3.3]{sinn2020}}]
\makelabel{satz:main:ueb:2:ex:1}
Sei $V$ ein Vektorraum über $\reell$ wie $\reell^{n}$ für ein $n\in\ntrlpos$.
Seien $\mathbf{v},\mathbf{w}\in V$ mit $\mathbf{v}\neq \mathbf{w}$ und $\mathbf{w}\neq\zerovector$
und sei
\begin{mathe}[mc]{rcl}
L &:= &\{s\mathbf{v} + (1-s)\mathbf{w}\mid s\in\reell\}\\
\end{mathe}
die Verbindungsgerade zw. $\mathbf{v}$ und $\mathbf{w}$.
Dann gilt $\zerovector\in L$ $\Leftrightarrow$ $\exists{c\in\reell:~}\mathbf{v}=c\mathbf{w}$.
\end{satz}
\end{schattierteboxdunn}
\begin{proof}
Der Beweis wird in zwei Teilen gezeigt.
\hinRichtung Angenommen, $\zerovector\in L$.
\textbf{Zu zeigen:} $\exists{c\in\reell:~}\mathbf{v}=c\mathbf{w}$.\\
Per Definition von $L$ existiert ein $s\in\reell$, so dass sich $\zerovector$
als $\zerovector=s\mathbf{v} + (1-s)\mathbf{w}$
darstellen lässt.
Daraus lässt sich ableiten:
\begin{mathe}[mc]{rcl}
\zerovector=s\mathbf{v} + (1-s)\mathbf{w}
&\Longleftrightarrow
&s\mathbf{v} = (s-1)\mathbf{w}\\
&\Longleftrightarrow
&\underbrace{%
(s=0\,\text{und}\,\mathbf{w}=s(\mathbf{w}-\mathbf{v})=\zerovector)
}_{%
\text{unmöglich, da $\mathbf{w}\neq\zerovector$ per Voraussetzung}
}
\,\text{oder}\,(s\neq 0\,\text{und}\,\mathbf{v} = ((s-1)/s)\mathbf{w})\\
&\Longleftrightarrow
&s\neq 0\,\text{und}\,\mathbf{v} = ((s-1)/s)\mathbf{w}\\
&\Longrightarrow
&\exists{c\in\reell:~}\mathbf{v} = c\mathbf{w}.\\
\end{mathe}
\herRichtung Angenommen, $\mathbf{v} = c\mathbf{w}$ für ein $c\in\reell$.
\textbf{Zu zeigen:} $\zerovector\in L$.\\
Per Voraussetzung gilt nun $\mathbf{v}\neq\mathbf{w}$, sodass $c=1$ direkt ausgeschlossen ist.\\
Setze nun \fbox{$s:=\frac{1}{1-c}\in\reell$}, was wohldefiniert ist, da $c\neq 1$.\\
Man berechnet nun
\begin{mathe}[mc]{rcccccccl}
\overbrace{s\mathbf{v}+(1-s)\mathbf{w}}^{\in L,\,\text{per Definition}}
&= &\frac{1}{1-c}c\mathbf{w}+(1-\frac{1}{1-c})\mathbf{w}
&= &(\underbrace{\frac{c}{1-c}+1-\frac{1}{1-c}}_{=\frac{c-1}{1-c}+1=0})\mathbf{w}
&= &0\mathbf{w}
&= &\zerovector.\\
\end{mathe}
Darum gilt $\zerovector\in L$.
\end{proof}
%% AUFGABE 2-2
\clearpage
\let\altsectionname\sectionname
\def\sectionname{Aufgabe}
\section[Aufgabe 2]{}
\label{ueb:2:ex:2}
\let\sectionname\altsectionname
\begin{enumerate}{\bfseries (a)}
%% AUFGABE 2-2a
\item
\begin{schattierteboxdunn}
\begin{satz}
\makelabel{satz:main:ueb:2:ex:2a}
Seien $\mathbf{v},\mathbf{v}^{\prime},\mathbf{w},\mathbf{w}^{\prime}\in\reell^{2}$
mit $\mathbf{w},\mathbf{w}^{\prime}\neq\zerovector$.
Seien
$L:=\{\mathbf{v}+t\mathbf{w}\mid t\in\reell\}$
und
$L^{\prime}:=\{\mathbf{v}^{\prime}+s\mathbf{w}^{\prime}\mid s\in\reell\}$.
Angenommen, $L\neq L^{\prime}$.
Dann sind folgende Aussagen äquivalent:
\begin{kompaktenum}{(i)}
\item\punktlabel{1}
$L\cap L^{\prime}=\leer$;
\item\punktlabel{2}
$\mathbf{w},\mathbf{w}^{\prime}$ sind kolinear,
d.\,h.
$\exists{c\in\reell:~}\mathbf{w}=c\mathbf{w}^{\prime}$.
\end{kompaktenum}
\nvraum{1}
\end{satz}
\end{schattierteboxdunn}
\begin{proof}
Der Beweis wird in zwei Teilen gezeigt.
\hinRichtung{1}{2} Angenommen, $L\cap L^{\prime}=\leer$.
\textbf{Zu zeigen:} $\exists{c\in\reell:~}\mathbf{w}=c\mathbf{w}^{\prime}$.\\
\fbox{Angenommen, dies sei nicht der Fall.}\\
Da $\mathbf{w},\mathbf{w}^{\prime}\neq\zerovector$ bedeutet dies,
dass $\mathbf{w},\mathbf{w}^{\prime}$ \emph{linear unabhängig} sind. ($\to$ Warum??)\\
Also gilt für den Untervektorraum
$U:=\span\{\mathbf{w},\mathbf{w}^{\prime}\}$,
dass $\dim(U)=2$.\\
Da $U\subseteq\reell^{2}$ Vektorräume sind und $\dim(U)=2=\dim(\reell^{2})$,
folgt hieraus, dass $U=\reell^{2}$. ($\to$ Warum??)\\
Betrachte bspw. den Vektor
\begin{mathe}[mc]{rcl}
\eqtag[eq:1-2:1:\beweislabel]
\mathbf{\xi} &:= &\mathbf{v}^{\prime}-\mathbf{v}\in\reell^{2}.\\
\end{mathe}
Dann $\mathbf{\xi}\in U=\span\{\mathbf{w},\mathbf{w}^{\prime}\}$.
Folglich existieren Skalare $\alpha,\beta\in\reell$,
so dass $\alpha\mathbf{w}+\beta\mathbf{w}^{\prime}=\mathbf{\xi}$
gilt.\\
Setze nun \fbox{$t:=\alpha$} und \fbox{$s:=-\beta$}.
Dann gilt
\begin{mathe}[mc]{rclcl}
\overbrace{%
\mathbf{v}+t\mathbf{w}
}^{\in L}
&= &(\mathbf{v}+t\mathbf{w})-(\mathbf{v}^{\prime}+s\mathbf{w}^{\prime})
+\mathbf{v}^{\prime}+s\mathbf{w}^{\prime}\\
&= &(\mathbf{v}-\mathbf{v}^{\prime})+(t\mathbf{w}-s\mathbf{w}^{\prime})
+\mathbf{v}^{\prime}+s\mathbf{w}^{\prime}\\
&= &(\mathbf{v}-\mathbf{v}^{\prime})+(\alpha\mathbf{w}+\beta\mathbf{w}^{\prime})
+\mathbf{v}^{\prime}+s\mathbf{w}^{\prime}\\
&\eqcrefoverset{eq:1-2:1:\beweislabel}{=}
&-\mathbf{\xi}+\mathbf{\xi}
+\mathbf{v}^{\prime}+s\mathbf{w}^{\prime}
&= &\underbrace{%
\mathbf{v}^{\prime}+s\mathbf{w}^{\prime}%
}_{\in L^{\prime}}.\\
\end{mathe}
Darum gilt $L\cap L^{\prime}\neq\leer$,
was ein Widerspruch ist.\\
Darum stimmt die o.\,s. Annahme nicht.
Also sind $\mathbf{w},\mathbf{w}^{\prime}$ kolinear.
\hinRichtung{2}{1} Angenommen, $\mathbf{w}=c\mathbf{w}^{\prime}$ für ein $c\in\reell$.
\textbf{Zu zeigen:} $L\cap L^{\prime}=\leer$.\\
\fbox{Angenommen, dies sei nicht der Fall.}
Dann existiert ein Vektor, $\mathbf{u}\in L\cap L^{\prime}$.\\
Per Konstruktion existieren dann $s_{0},t_{0}\in\reell$,
so dass
\begin{mathe}[mc]{rcccl}
\mathbf{v}+t_{0}\mathbf{w} &= &\mathbf{u} &= &\mathbf{v}^{\prime}+s_{0}\mathbf{w}^{\prime}.\\
\end{mathe}
Aus der Voraussetzung für diese Richtung folgt
\begin{mathe}[mc]{rcl}
\eqtag[eq:2-1:1:\beweislabel]
\mathbf{v}^{\prime} &= &\mathbf{v}+(t_{0}-s_{0}c)\mathbf{w}\\
\end{mathe}
Beachte, dass \fbox{$c\neq 0$}, denn sonst würde $\mathbf{w}=c\mathbf{w}^{\prime}=\zerovector$ gelten,
was ein Widerspruch ist. Wir berechnen
\begin{mathe}[mc]{rcl}
\eqtag[eq:2-1:2:\beweislabel]
L^{\prime} &= &\{\mathbf{v}^{\prime}+s\mathbf{w}^{\prime}\mid s\in\reell\}\\
&\eqcrefoverset{eq:2-1:1:\beweislabel}{=}
&\{\mathbf{v}+(t_{0}-s_{0}c)\mathbf{w}+sc\mathbf{w}\mid s\in\reell\}\\
&= &\{\mathbf{v}+(t_{0}+(s-s_{0})c)\mathbf{w}\mid s\in\reell\}\\
&= &\{\mathbf{v}+t\mathbf{w}\mid t\in R\},\\
\end{mathe}
wobei $R=\{t_{0}+(s-s_{0})c\mid s\in\reell\}=f(\reell)$.
Also $R=f(\reell)$, wobei ${f:\reell\to\reell}$ eine durch ${f(s)=t_{0}+(s-s_{0})c}$ definierte Funktion ist.
Da $c\neq 0$, ist es einfach zu sehen, dass $f$ surjektiv ist (in der Tat bijektiv).
Darum gilt $R=f(\reell)=\reell$.\\
Aus \eqcref{eq:2-1:2:\beweislabel} folgt also
${L^{\prime}=\{\mathbf{v}+t\mathbf{w}\mid t\in\reell\}=L}$,
was ein Widerspruch ist.\\
Darum stimmt die o.\,s. Annahme nicht.
Also gilt $L\cap L^{\prime}=\leer$.
\end{proof}
%% AUFGABE 2-2b
\item
Wir zeigen nun ein minimales Beispiel dafür, dass \Cref{satz:main:ueb:2:ex:2a}
im allgemeinen für andere Vektorräume nicht gilt.
Betrachte den Vektorraum $\reell^{3}$.
Betrachte die folgenden Vektoren in $\reell^{3}$:
\begin{mathe}[mc]{rclqrclqrclqrcl}
\mathbf{v} &= &\begin{svector}0\\0\\0\\\end{svector},
&\mathbf{v}^{\prime} &= &\begin{svector}1\\0\\0\\\end{svector},
&\mathbf{w} &= &\begin{svector}0\\1\\0\\\end{svector},
&\mathbf{w}^{\prime} &= &\begin{svector}0\\1\\1\\\end{svector}.\\
\end{mathe}
Bis auf 2-Dimensionalität erfüllen diese die Voraussetzungen in \Cref{satz:main:ueb:2:ex:2a}.
Einerseits wurden $\mathbf{w}$, $\mathbf{w}^{\prime}$ so gewählt, dass sie \emph{nicht} kolinear sind.
Dennoch schneiden sich die beiden Geraden, $L$, $L^{\prime}$, nicht,
da
${L\subseteq \{\mathbf{x}\in\reell^{3}\mid x_{1}=0\}=:E}$
und
${L^{\prime}\subseteq \{\mathbf{x}\in\reell^{3}\mid x_{1}=1\}=:E^{\prime}}$
und offensichtlich $E\cap E'=\leer$.
\end{enumerate}
%% AUFGABE 2-3
\clearpage
\let\altsectionname\sectionname
\def\sectionname{Aufgabe}
\section[Aufgabe 3]{}
\label{ueb:2:ex:3}
\let\sectionname\altsectionname
\begin{enumerate}{\bfseries (a)}
%% AUFGABE 2-3a
\item
Für jedes $\gamma\in\reell$ sei die Gerade $L_{\gamma}\subseteq\reell^{2}$ gegeben durch
\begin{mathe}[mc]{rcl}
L_{\gamma} &= &\{(x,y)\in\reell^{2}\mid 2x+y=\gamma\cdot(x-3y-7)\}.\\
\end{mathe}
\begin{schattierteboxdunn}
\begin{satz}
\makelabel{satz:main:ueb:2:ex:3a}
Es gibt exakt einen Punkt in dem Schnitt aus den Geraden, $L_{\gamma}$, $\gamma\in\reell$.
Es gilt nämlich ${\displaystyle\bigcap_{\gamma\in\reell}L_{\gamma}=\{\mathbf{\xi}\}}$,
wobei $\mathbf{\xi}=(1,-2)$.
\end{satz}
\end{schattierteboxdunn}
\begin{proof}
Wir teilen diesen Beweis in zwei Teilen auf:
\BeweisRichtung[$\supseteq$]
Es reicht aus, für alle $\gamma\in\reell$ \textbf{zu zeigen}, dass $\mathbf{\xi}\in L_{\gamma}$.\\
Fixiere also ein beliebiges $\gamma\in\reell$. Dann
\begin{mathe}[mc]{rclclcll}
2\xi_{1}+\xi_{2}
&= &2\cdot 1+(-2)
&= &0,
&&&\text{und}\\
\gamma\cdot(\xi_{1}-3\xi_{2}-7)
&= &\gamma\cdot(1-3(-2)-7)
&= &\gamma\cdot 0
&= &0.\\
\end{mathe}
Also ${2\xi_{1}+\xi_{2}=\gamma\cdot(\xi_{1}-3\xi_{2}-7)}$.
Folglich gilt $\mathbf{\xi}\in L_{\gamma}$ per Konstruktion.
\BeweisRichtung[$\subseteq$]
Sei ${\mathbf{\eta}:=(x,y)\in\bigcap_{\gamma\in\reell}L_{\gamma}}$ beliebig.
\textbf{Zu zeigen:} $\mathbf{\eta}=\mathbf{\xi}$.\\
Zu diesem Zwecke seien $\gamma_{1},\gamma_{2}\in\reell$ irgendwelche Werte mit $\gamma_{1}\neq\gamma_{2}$.
Per Wahl gilt $\mathbf{\eta}\in L_{\gamma_{1}}\cap L_{\gamma_{2}}$.
Also
\begin{mathe}[mc]{rcl}
2x+y &= &\gamma_{1}\cdot(x-3x-7),\,\text{und}\\
2x+y &= &\gamma_{2}\cdot(x-3x-7).\\
\end{mathe}
Wir können ganz naiv arbeiten und die Gleichungen subtrahieren.
Dies liefert
$(\gamma_{1}-\gamma_{2})\cdot(x-3x-7)=0$,
woraus sich ergibt, dass
$x-3y-7=0$
gelten muss, da $\gamma_{1}\neq\gamma_{2}$.
Eingesetzt in die erste Gleichung oben liefert
$2x+y=\gamma\cdot 0=0$.
Darum muss $\begin{svector}x\\y\\\end{svector}$
das LGS $(A|\mathbf{b})$ lösen, wobei
\begin{mathe}[mc]{rclqrcl}
A &= &\begin{smatrix}
1&-3\\
2&1\\
\end{smatrix},
&\mathbf{b} &= &\begin{svector}7\\0\\\end{svector}
\end{mathe}
\begin{algorithm}[\rtab][\rtab]
Gaußverfahren angewandt auf $(A|\mathbf{b})$:
\begin{mathe}[mc]{c}
\begin{matrix}{cc|c}
1 &-3 &7\\
2 &1 &0\\
\end{matrix}\\
\end{mathe}
Wende die Zeilentransformation
${Z_{2}\leftsquigarrow Z_{2}-2\cdot Z_{1}}$
an:
\begin{mathe}[mc]{c}
\begin{matrix}{cc|c}
1 &-3 &7\\
0 &7 &-14\\
\end{matrix}\\
\end{mathe}
Aus der Stufenform erschließt sich
\begin{mathe}[bc]{rclcl}
y &= &\frac{-14}{7} &= &-2\\
x &= &7 + 3\cdot y &= &1.\\
\end{mathe}
\end{algorithm}
Also
${\mathbf{\eta}=(x, y)=(1, -2)=\mathbf{\xi}}$
für alle $\mathbf{\eta}\in\bigcap_{\gamma\in\reell}L_{\gamma}$.
Das heißt $\bigcap_{\gamma\in\reell}L_{\gamma}\subseteq\{\mathbf{\xi}\}$.
\end{proof}
\clearpage
%% AUFGABE 2-3b
\item
\begin{enumerate}{\bfseries (i)}
%% AUFGABE 2-3b-i
\item
Sei $\gamma\in\reell$. Dann gilt
\begin{mathe}[mc]{rcl}
(-3,2)\in L_{\gamma}
&\Longleftrightarrow
&2(-3)+(2)=\gamma\cdot((-3)-3(2)-7)\\
&\Longleftrightarrow
&\gamma=\frac{-4}{-16}=\frac{1}{4}.\\
\end{mathe}
Also ist \fbox{$\gamma=\frac{1}{4}$} der eindeutige Parameter,
für den $(-3,2)\in L_{\gamma}$ gilt.
%% AUFGABE 2-3b-ii
\item
Sei $\gamma\in\reell$. Man beobachte, dass
\begin{longmathe}[mc]{RCL}
L_{\gamma}
&= &\{(x,y)\in\reell^{2}\mid (2-\gamma)x+(1+3\gamma)y=-7\gamma\}\\
&= &\begin{cases}[m]{lcl}
\{(x,y)\in\reell^{2}\mid 0x + (1+3\cdot 2)y=-7\cdot 2\}
&: &\gamma=2\\
\{(x,y)\in\reell^{2}\mid (2-\frac{-1}{3})x + 0y=-7\cdot\frac{-1}{3}\}
&: &\gamma=-\frac{1}{3}\\
\{(x,y)\in\reell^{2}\mid (2-\gamma)x+(1+3\gamma)y=-7\gamma\}
&: &\text{sonst}
\end{cases}\\
&= &\begin{cases}[m]{lcl}
\{(x,y)\in\reell^{2}\mid y=-2\}
&: &\gamma=2\\
\{(x,y)\in\reell^{2}\mid x=1\}
&: &\gamma=-\frac{1}{3}\\
\{(x,y)\in\reell^{2}\mid y=\frac{\gamma-2}{1+3\gamma}x - \frac{7\gamma}{1+3\gamma}\}
&: &\text{sonst}
\end{cases}.\\
\end{longmathe}
Daraus folgt, dass $L_{\gamma}$
\begin{kompaktitem}
\item
parallel zur $x$-Achse für $\gamma=2$ ist,
\item
parallel zur $y$-Achse für $\gamma=-\frac{1}{3}$ ist,
\item
und ansonsten weder zur $x$- noch $y$-Achse parallel ist,
da in diesem Falle $L_{\gamma}$ die Gerade »${y=ax+b}$« ist, wobei $a\neq 0$.
\end{kompaktitem}
Also ist der gesuchte Parameterwert eindeutig \fbox{$\gamma=-\frac{1}{3}$}.
%% AUFGABE 2-3b-iii
\item
Die Gerade »$x-2y=-1$« lässt sich äquivalent
als »$y=\frac{1}{2}x+1$
darstellen.
Darum wird ein Wert $\gamma\in\reell$ gesucht,
so dass die Gerade $L_{\gamma}$ weder zur $x$- noch $y$-Achse parallel ist,
und die die $y$-$x$-Steigung $\frac{1}{2}$ hat.
Nach der o.\,s. Berechnung in (ii) kommt dies nur für den 3. Fall in Frage.
Darum gilt
\begin{mathe}[mc]{rcl}
L_{\gamma}\,\text{parallel zur Gerade »$x-2y=-1$«}
&\Longleftrightarrow
&\gamma\notin\{2,-\frac{1}{3}\}
\,\text{und}\,
\frac{\gamma-2}{1+3\gamma}=\frac{1}{2}\\
&\Longleftrightarrow
&\gamma\notin\{2,-\frac{1}{3}\}
\,\text{und}\,
(\gamma-2)=\frac{1}{2}(1+3\gamma)\\
&\Longleftrightarrow
&\gamma\notin\{2,-\frac{1}{3}\}
\,\text{und}\,
\gamma=-5\\
&\Longleftrightarrow
&\gamma=-5.\\
\end{mathe}
Also ist der gesuchte Parameterwert eindeutig \fbox{$\gamma=-5$}.
\end{enumerate}
\end{enumerate}
%% ********************************************************************************
%% FILE: body/uebung/ueb3.tex
%% ********************************************************************************
\setcounternach{chapter}{3}
\chapter[Woche 3]{Woche 3}
\label{ueb:2}
\textbf{ACHTUNG.}
Diese Lösungen dienen \emph{nicht} als Musterlösungen sondern eher als Referenz.
Hier wird eingehender gearbeitet, als generell verlangt wird.
Das Hauptziel hier ist, eine Variant anzubieten, gegen die man seine Versuche vergleichen kann.
%% AUFGABE 3-1
\let\altsectionname\sectionname
\def\sectionname{Aufgabe}
\section[Aufgabe 1]{}
\label{ueb:3:ex:1}
\let\sectionname\altsectionname
Wir arbeiten im Vektorraum $\reell^{3}$ und betrachten die Vektoren
\begin{mathe}[mc]{rclqrclqrclqrcl}
\mathbf{v}_{1} &= &\begin{svector}1\\3\\1\\\end{svector}
&\mathbf{v}_{2} &= &\begin{svector}-2\\5\\-2\\\end{svector}
&\mathbf{w}_{1} &= &\begin{svector}4\\-3\\-3\\\end{svector}
&\mathbf{w}_{2} &= &\begin{svector}0\\1\\1\\\end{svector}\\
\end{mathe}
\textbf{Zu berechnen:}
$U:=\span\{\mathbf{v}_{1},\mathbf{v}_{2}\}
\cap\span\{\mathbf{w}_{1},\mathbf{w}_{2}\}$
als Untervektorraum von $\reell^{3}$.\\
Zu diesem Zwecke betrachte einen beliebigen Vektor, $\mathbf{\xi}\in\reell^{3}$.
Es gilt
\begin{mathe}[mc]{rcl}
\eqtag[eq:0:ueb:3:ex:1]
\mathbf{\xi}\in U
&\Longleftrightarrow
&\exists{t_{1},t_{2},t_{3},t_{4}\in\reell:~}
\mathbf{\xi}=t_{1}\mathbf{v}_{1}+t_{2}\mathbf{v}_{2}
\,\text{und}\,
\mathbf{\xi}=t_{3}\mathbf{w}_{1}+t_{4}\mathbf{w}_{2}\\
&\Longleftrightarrow
&\exists{\mathbf{t}\in\reell^{4}:~}
\mathbf{\xi}=t_{1}\mathbf{v}_{1}+t_{2}\mathbf{v}_{2}
\,\text{und}\,
t_{1}\mathbf{v}_{1}+t_{2}\mathbf{v}_{2}
=t_{3}\mathbf{w}_{1}+t_{4}\mathbf{w}_{2}\\
&\Longleftrightarrow
&\exists{\mathbf{t}\in\reell^{4}:~}
\mathbf{\xi}=t_{1}\mathbf{v}_{1}+t_{2}\mathbf{v}_{2}
\,\text{und}\,
t_{1}\mathbf{v}_{1}+t_{2}\mathbf{v}_{2}
-t_{3}\mathbf{w}_{1}-t_{4}\mathbf{w}_{2}
=\zerovector\\
&\Longleftrightarrow
&\exists{\mathbf{t}\in\reell^{4}:~}
\mathbf{\xi}=t_{1}\mathbf{v}_{1}+t_{2}\mathbf{v}_{2}
\,\text{und}\,
t_{1}\mathbf{v}_{1}+t_{2}\mathbf{v}_{2}
+t_{3}\mathbf{w}_{1}+t_{4}\mathbf{w}_{2}
=\zerovector\\
&\Longleftrightarrow
&\exists{\mathbf{t}\in\reell^{4}:~}
\mathbf{\xi}=t_{1}\mathbf{v}_{1}+t_{2}\mathbf{v}_{2}
\,\text{und}\,
A\mathbf{t}=\zerovector,\\
\end{mathe}
wobei
\begin{mathe}[mc]{rcccl}
A &:= &\left(
\mathbf{v}_{1}~
\mathbf{v}_{2}~
\mathbf{w}_{1}~
\mathbf{w}_{2}
\right)
&= &\begin{smatrix}
1&-2&4&0\\
3&5&-3&1\\
1&-2&-3&1\\
\end{smatrix}\\
\end{mathe}
Darum ist es notwendig und hinreichend,
die \emph{homogenen Lösungen} für $A$ zu finden,
und daraus die Parameter abzulesen.
\begin{algorithm}[\rtab][\rtab]
Homogenes Problem für $A$:\\
Zeilentransformationen
${Z_{2}\leftsquigarrow Z_{2}-3\cdot Z_{1}}$,
${Z_{3}\leftsquigarrow Z_{3}-Z_{1}}$
anwenden:
\begin{mathe}[mc]{c}
\begin{smatrix}
1&-2&4&0\\
0&11&-15&1\\
0&0&-7&1\\
\end{smatrix}\\
\end{mathe}
Wende die Zeilentransformation
${Z_{2}\leftsquigarrow Z_{2}-Z_{3}}$
an:
\begin{mathe}[mc]{c}
\begin{smatrix}
1&-2&4&0\\
0&11&-8&0\\
0&0&-7&1\\
\end{smatrix}\\
\end{mathe}
Aus der Zeilenstufenform erschließt sich, dass $t_{4}$ frei ist.
Also $t_{4}=\alpha$ für ein frei wählbares $\alpha\in\reell$.
Aus der Stufenform von Gleichungen $3,2,1$ erschließt sich
\begin{mathe}[mc]{rcl}
t_{3} &= &\frac{1}{7}t_{4} = \frac{1}{7}\alpha\\
t_{2} &= &\frac{8}{11}t_{3} = \frac{8}{77}\alpha\\
t_{1} &= &2t_{2} - 4t_{3}
= \frac{16}{77}\alpha - \frac{4}{7}\alpha
= -\frac{28}{77}\alpha\\
\end{mathe}
Man kann o.\,E. $\alpha$ durch $\beta:=-77\alpha$ ersetzen.
Also ist die homogene Lösung gegeben durch
\begin{mathe}[mc]{rcl}
\mathbf{t} &= &\beta\begin{svector}28\\-8\\-11\\-77\\\end{svector},
\quad\text{mit $\beta\in\reell$ frei wählbar}.
\end{mathe}
\end{algorithm}
Wir können nun \eqcref{eq:0:ueb:3:ex:1} fortsetzen und erhalten
\begin{mathe}[mc]{rcl}
\eqtag[eq:1:ueb:3:ex:1]
\mathbf{\xi}\in U
&\Longleftrightarrow
&\exists{\mathbf{t}\in\reell^{4}:~}
\mathbf{\xi}=t_{1}\mathbf{v}_{1}+t_{2}\mathbf{v}_{2}
\,\text{und}\,
A\mathbf{t}=\zerovector\\
&\Longleftrightarrow
&\exists{\mathbf{t}\in\reell^{4}:~}
\mathbf{\xi}=t_{1}\mathbf{v}_{1}+t_{2}\mathbf{v}_{2}
\,\text{und}\,
\exists{\beta\in\reell:~}
\mathbf{t}=\beta\begin{svector}28\\-8\\-11\\-77\\\end{svector}\\
&\Longleftrightarrow
&\exists{\beta\in\reell:~}
\mathbf{\xi}=\beta\cdot(
\underbrace{
28\mathbf{v}_{1}+-8\mathbf{v}_{2}
}_{=:\mathbf{u}}
)\\
&\Longleftrightarrow &\mathbf{\xi}\in\span\{\mathbf{u}\}\\
\end{mathe}
für alle $\mathbf{\xi}\in\reell^{3}$.\\
Es gilt
\begin{mathe}[mc]{rcccccl}
\mathbf{u}
&= &28\begin{svector}1\\3\\1\\\end{svector}
-8\begin{svector}-2\\5\\-2\\\end{svector}
&= &\begin{svector}44\\44\\44\\\end{svector}
&= &44\begin{svector}1\\1\\1\\\end{svector}.\\
\end{mathe}
Aus \eqcref{eq:1:ueb:3:ex:1} ergibt sich der zu berechnende Untervektorraum
als
\begin{mathe}[mc]{rcccccccl}
\span\{\mathbf{v}_{1},\mathbf{v}_{2}\}
\cap\span\{\mathbf{w}_{1},\mathbf{w}_{2}\}
&= &U
&= &\span\{\mathbf{u}\}
&= &\span\{44\begin{svector}1\\1\\1\\\end{svector}\}
&= &\span\{\begin{svector}1\\1\\1\\\end{svector}\}.\\
\end{mathe}
%% AUFGABE 3-2
\clearpage
\let\altsectionname\sectionname
\def\sectionname{Aufgabe}
\section[Aufgabe 2]{}
\label{ueb:3:ex:2}
\let\sectionname\altsectionname
Seien $X$, $Y$ nicht leere Mengen und ${f:X\to Y}$ eine Funktion.
\begin{enumerate}{\bfseries (a)}
%% AUFGABE 3-2a
\item
\begin{claim*}
Die Aussage $\forall{A,B\subseteq X:~}f(A\cap B)=f(A)\cap f(B)$
ist \fbox{\uline{nicht} allgemein gültig}.
\end{claim*}
\begin{proof}
Betrachte das Beispiel $X=\{0,1\}$, $Y=\{2\}$, und ${f:X\to Y}$ mit $f(x)=2$ für alle $x\in X$.
Für $A=\{0\}$ und $B=\{1\}$
gilt $f(A\cap B)=f(\leer)=\leer$,
während $f(A)\cap f(B)=\{2\}\cap\{2\}=\{2\}$.
Also $f(A\cap B)\neq f(A)\cap f(B)$.
Darum ist dies ein Gegenbeispiel zur Aussage.
\end{proof}
\text{Bemerkung.} Die Aussage ist eigentlich genau dann wahr, wenn $f$ injektiv ist.
%% AUFGABE 3-2b
\item
\begin{claim*}
Die Aussage $\forall{A,B\subseteq X:~}f(A\cup B)=f(A)\cup f(B)$
ist \fbox{allgemein gültig}.
\end{claim*}
Für manche (doppelte) Implikationen hier, nämlich für den Umgang mit Existenzquantoren,
braucht man Grundkenntnisse in Prädikatenlogik 1. Stufe.
Hierfüg gibt es zahlreiche Einführungswerke in die mathematische Logik,
bspw. \cite{ebbinghaus2018}.
\begin{proof}
Seien $A,B\subseteq X$ beliebige Teilmengen.
Es reicht aus \textbf{zu zeigen},
dass $y\in f(A\cup B)\Leftrightarrow y\in f(A)\cup f(B)$
für alle $y\in Y$ gilt.\\
Sei also $y\in Y$ beliebig. Es gilt
\begin{longmathe}[mc]{RCL}
y\in f(A\cup B)
&\Longleftrightarrow
&\exists{x\in A\cup B:~}y=f(x)\\
&\Longleftrightarrow
&\exists{x\in X:~}x\in A\cup B\,\text{und}\,y=f(x)\\
&\Longleftrightarrow
&\exists{x\in X:~}
(x\in A\,\text{oder}\,x\in B)
\,\text{und}\,y=f(x)\\
&\Longleftrightarrow
&\exists{x\in X:~}
\big(
(x\in A\,\text{und}\,y=f(x))
\,\text{oder}\,
(x\in B\,\text{und}\,y=f(x))
\big)\\
&\Longleftrightarrow
&\exists{x\in X:~}(x\in A\,\text{und}\,y=f(x))
\,\text{oder}\,
\exists{x\in X:~}(x\in B\,\text{und}\,y=f(x))\\
&\Longleftrightarrow
&\exists{x\in A:~}y=f(x)
\,\text{oder}\,
\exists{x\in B:~}y=f(x)\\
&\Longleftrightarrow
&y\in f(A)\,\text{oder}\,y\in f(B)\\
&\Longleftrightarrow
&y\in f(A)\cup f(B).\\
\end{longmathe}
Darum gilt $f(A\cup B)=f(A)\cup f(B)$ für alle $A,B\subseteq X$.
\end{proof}
%% AUFGABE 3-2c
\item
\begin{claim*}
Die Aussage $\forall{A\subseteq X:~}f(X\ohne A)=Y\ohne f(A)$
ist \fbox{\uline{nicht} allgemein gültig}.
\end{claim*}
\begin{proof}
Betrachte das Beispiel $X=\{0,1\}$, $Y=\{2\}$, und ${f:X\to Y}$ mit $f(x)=2$ für alle $x\in X$.
Für $A=\{0\}$
gilt $f(X\ohne A)=f(\{1\})=\{2\}$,
während $Y\cap f(A)=\{2\}\ohne\{2\}=\leer$.
Also $f(X\ohne A)\neq Y\cap f(A)$.
Darum ist dies ein Gegenbeispiel zur Aussage.
\end{proof}
\text{Bemerkung.} Die Aussage ist eigentlich genau dann wahr, wenn $f$ bijektiv ist.
Und eine leicht modifizierte Aussage,
$\forall{A\subseteq X:~}f(X\ohne A)=f(X)\cap f(A)$,
ist genau dann wahr, wenn $f$ injektiv ist.
%% AUFGABE 3-2d
\item
\begin{claim*}
Die Aussage $\forall{A,B\subseteq Y:~}f^{-1}(A\cap B)=f^{-1}(A)\cap f^{-1}(B)$
ist \fbox{allgemein gültig}.
\end{claim*}
\begin{proof}
Seien $A,B\subseteq Y$ beliebige Teilmengen.
Es reicht aus \textbf{zu zeigen},
dass $x\in f^{-1}(A\cap B)\Leftrightarrow x\in f^{-1}(A)\cap f^{-1}(B)$
für alle $x\in X$ gilt.\\
Sei also $x\in X$ beliebig. Es gilt
\begin{longmathe}[mc]{RCL}
x\in f^{-1}(A\cap B)
&\Longleftrightarrow
&f(x)\in A\cap B\\
&\Longleftrightarrow
&f(x)\in A\,\text{und}\,f(x)\in B\\
&\Longleftrightarrow
&x\in f^{-1}(A)\,\text{und}\,x\in f^{-1}(B)\\
&\Longleftrightarrow
&x\in f^{-1}(A)\cap f^{-1}(B).\\
\end{longmathe}
Darum gilt $f^{-1}(A\cap B)=f^{-1}(A)\cap f^{-1}(B)$ für alle $A,B\subseteq Y$.
\end{proof}
%% AUFGABE 3-2e
\item
\begin{claim*}
Die Aussage $\forall{A,B\subseteq Y:~}f^{-1}(A\cup B)=f^{-1}(A)\cup f^{-1}(B)$
ist \fbox{allgemein gültig}.
\end{claim*}
\begin{proof}
Seien $A,B\subseteq Y$ beliebige Teilmengen.
Es reicht aus \textbf{zu zeigen},
dass $x\in f^{-1}(A\cup B)\Leftrightarrow x\in f^{-1}(A)\cup f^{-1}(B)$
für alle $x\in X$ gilt.\\
Sei also $x\in X$ beliebig. Es gilt
\begin{longmathe}[mc]{RCL}
x\in f^{-1}(A\cup B)
&\Longleftrightarrow
&f(x)\in A\cup B\\
&\Longleftrightarrow
&f(x)\in A\,\text{oder}\,f(x)\in B\\
&\Longleftrightarrow
&x\in f^{-1}(A)\,\text{oder}\,x\in f^{-1}(B)\\
&\Longleftrightarrow
&x\in f^{-1}(A)\cup f^{-1}(B).\\
\end{longmathe}
Darum gilt $f^{-1}(A\cup B)=f^{-1}(A)\cup f^{-1}(B)$ für alle $A,B\subseteq Y$.
\end{proof}
\end{enumerate}
%% AUFGABE 3-3
\clearpage
\let\altsectionname\sectionname
\def\sectionname{Aufgabe}
\section[Aufgabe 3]{}
\label{ueb:3:ex:3}
\let\sectionname\altsectionname
\begin{enumerate}{\bfseries (a)}
%% AUFGABE 3-3a
\item
Seien $n\in\ntrlpos$ und $v\in\reell^{n}$.
Sei ${f:\reell^{n}\to\reell^{n}}$ durch $f(x)=x+v$ definiert.
\begin{claim*}
$f$ ist \fbox{bijektiv}.
\end{claim*}
\begin{proof}
Sei ${g:\reell^{n}\to\reell^{n}}$ durch $g(x)=x-v$ definiert.
Es ist einfach zu sehen,
dass $f\circ g=\id_{\reell^{n}}$
und $g\circ f=\id_{\reell^{n}}$.
Per Definition ist als $f$ eine Bijektion mit Inversem $g$.
\end{proof}
%% AUFGABE 3-3b
\item
Seien $n\in\ntrlpos$ und $X=\reell^{n}\times(\reell^{n}\ohne\{\zerovector\}$.
Sei $Y$ die Menge aller Geraden im $\reell^{n}$.
Sei ${f:X\to Y}$ durch $f(v,w)=\{v+t\cdot w\mid t\in\reell\}$ definiert.
\begin{claim*}
$f$ ist \fbox{surjektiv} aber \fbox{nicht injektiv}.
\end{claim*}
\begin{proof}
\uwave{{\bfseries Surjektivität}}\\
\textbf{Idee:} Folgt aus der Definition von Geraden durch Parameter.\\
Sei $L\subseteq\reell^{n}$ eine beliebige Gerade. \textbf{Zu zeigen:} $L\in f(X)$.\\
Nun, \emph{per Definition} einer Geraden existieren
$u,v\in\reell^{n}$ mit $w\neq\zerovector$
und so dass $L=\{u+t\cdot w\mid t\in\reell\}$.
Offensichtlicht gilt $(v,w)\in X$.
Darum gilt $L=f((v,w))\in f(X)$.
\uwave{{\bfseries Nichtinjektivität}}\\
\textbf{Idee:} Wir wissen, dass verschiedene aber parallele Vektoren dieselbe Gerade definieren.\\
Fixiere beliebiges $v,w\in\reell^{n}$
und wähle ein $c\in\reell\ohne\{0,1\}$.\\
Dann sind $w,cw\neq\zerovector$ verschiedene aber parallele Vektoren.\\
Darum gilt $f((v,w))=\{v+t\cdot w\mid t\in\reell\}=\{v+tc\cdot w\mid t\in\reell\}=f((v,cw))$.\\
Da $(v,w)\neq(v,cw)$, ist $f$ somit nicht injektiv.
\end{proof}
%% AUFGABE 3-3c
\item
Es sei $X$ die Menge aller Bücher in einem fixierten Kontext.
Sei $Y$ die Menge alle Autor(inn)en von Büchern.
Sei ${f:X\to\Pot(Y)}$ definiert durch
$f(x)=\{y\mid \text{$y$ ein(e) Autor(in) vom Buch $x$}\}$
für alle $x\in X$.
\begin{claim*}
$f$ ist \fbox{nicht im Allgemeinen injektiv} und \fbox{niemals surjektiv}.
\end{claim*}
\begin{proof}
\uwave{{\bfseries Nichtsurjektivität}}\\
\textbf{Zu zeigen:} Es gibt konstellationen von Autor(inn)en, die kein gemeinsames Buch verfasst haben.\\
Es gibt \emph{immer} eine(n) Autor(in) eines Buchs,
sodass $\leer\notin f(X)$ in allen Kontexten.
Darum ist $f$ niemals surjektiv.
\uwave{{\bfseries Nichtinjektivität}}\\
\textbf{Zu zeigen:} Es gibt zwei verschiedene Bücher,
die von der gleichen Konstellation an Autor(inn)en
verfasst wurden.
In unserem Kontext hat bspw. $a=\text{{\itshape JK~Rowling}}$ alleine die Bücher
${b_{1}:=\text{{\itshape »HP~and~the~Philosopher's~Stone«}}}$
und
${b_{2}:=\text{{\itshape »HP~and~the~Goblet~of~Fire«}}}$
geschrieben.
Darum $b_{1}\neq b_{2}$ und $f(b_{1})=\{a\}=f(b_{2})$.
Also ist $f$ in unserem Kontext nicht injektiv.
\end{proof}
\textbf{Anmerkung.}
Falls wir $\leer$ von der Bildmenge $\Pot(Y)$ exludieren,
dann können wir mindestens dafür argumentieren,
dass $f$ \fbox{nicht im Allgemeinen surjektiv} ist:
In unserem konkreten Kontext haben bspw. {\itshape JK~Rowling} und {\itshape Oscar~Wilde} nie am selben Buch gearbeitet,
also gilt $\{\text{JK Rowling},\,\text{Oscar Wilde}\}\notin f(X)$.
In der Tat ist ein Kontext kaum vorstellbar,
in dem sich \emph{alle} Autor(inn)en an einem gemeinsamen Buch beteiligt haben,
d.\,h. $Y\in f(X)$ sowie alle „große“ Teilmengen sind fast immer ausgeschlossen.
%% AUFGABE 3-3d
\item
Seien $X$ die Menge aller in Deutschland zugelassener Kfz und
$Y$ die Menge aller amtlicher Kennzeichen.
Sei ${f:X\to Y}$ die Abbildung, die jedem Kfz sein Kennzeichen zuordnet.
\begin{claim*}
$f$ ist \fbox{injektiv} aber \fbox{nicht im Allgemeinen surjektiv}.
\end{claim*}
\begin{proof}
\uwave{{\bfseries Injektivität:}}
Jedes Kennzeichen darf per Gesetz nur einem Kfz zugehören.
\uwave{{\bfseries Nichtsurjektivität:}}
Es besteht zwar die Chance, dass irgendwann alle Kennzeichen aufgebraucht werden,
aber in der Praxis ist die Menge $Y$ sehr groß,
dass dies aktuell und für eine lange Zeit nicht vorkommt.
\end{proof}
\end{enumerate}
\setcounternach{part}{2}
\part{Selbstkontrollenaufgaben}
\def\chaptername{SKA Blatt}
%% ********************************************************************************
%% FILE: body/ska/ska4.tex
%% ********************************************************************************
\setcounternach{chapter}{4}
\chapter[Woche 4]{Woche 4}
\label{ska:4}
%% SKA 4-1
\let\altsectionname\sectionname
\def\sectionname{SKA}
\section[Aufgabe 1]{}
\label{ska:4:ex:1}
\let\sectionname\altsectionname
Seien $X$, $Y$ nicht leere Mengen.
Einer Abbildung, $f:X\to Y$,
können wir eindeutig die Relation
$\graph(f):=\{(x,y)\in X\times Y\mid f(x)=y\}$
zuordnen. Dies nennt sich der \textbf{Graph von $f$}
(siehe \cite[\S{}2.3]{sinn2020}---dort wird dies mit $\Gamma_{f}$ bezeichnet).
Hier ist $\graph(f)$ also eine Relation auf $X\times Y$.
In der Tat \emph{setzen} manche Werke Funktionen mit ihrem Graphen gleich
(siehe bspw. \cite[S.11]{jech1997}),
aber dies ist streng genommen nicht die ganze Wahrheit.
%% SKA 4-2
\let\altsectionname\sectionname
\def\sectionname{SKA}
\section[Aufgabe 2]{}
\label{ska:4:ex:2}
\let\sectionname\altsectionname
\textbf{Hinweis:} Hier scheint im Punkt (ii) etwas verwechselt worden zu sein.
Seien $M$, $N$ Mengen und $R\subseteq M\times N$.
\begin{claim}
\makelabel{claim:main:ska:4:ex:2}
Angenommen, $R$ erfülle folgende Eigenschaften:
\begin{kompaktenum}{\bfseries (i)}[\rtab][\rtab]
\item\punktlabel{1}
$\forall{x\in M:~}\exists{y\in N:~}(x,y)\in R$
\item\punktlabel{2}
$\forall{x\in M:~}\forall{y,y'\in N:~}
(x,y),(x,y')\in R
\Rightarrow y=y'$
\end{kompaktenum}
Dann existiert eine (notwendigerweise eindeutige) Funktion,
${f:M\to N}$,
so dass $\graph(f)=R$.
\end{claim}
\begin{einzug}[\rtab][\rtab]
\begin{proof}
Wir definieren ${f:M\to N}$ durch
\begin{mathe}[mc]{rcl}
f(x) &= &y\\
\end{mathe}
für $(x,y)\in R$.
Offensichtlich gilt
$\graph(f)
=\{(x,y)\in M\times N\mid f(x)=y\}
=\{(x,y)\in M\times N\mid (x,y)\in R\}
=R$.
\textbf{Zu zeigen:}
(1) $f$ ist überall definiert;
(2) $f$ ist wohldefiniert.
\begin{kompaktenum}[\rtab][\rtab]
\item[\uwave{{\bfseries Überall definiert:}}]
Sei $x\in M$.
\textbf{Zu zeigen:} $f(x)=y$ für ein $y\in N$.\\
Eigenschaft \punktlabel{1} besagt, dass ein $y\in M$ existiert,
so dass $(x,y)\in R$.
Per Konstruktion erhalten wir, dass $f(x)=y$ gilt.
\item[\uwave{{\bfseries Wohldefiniertheit:}}]
Seien $x\in M$ und $y,y'\in N$.
Angenommen, $f(x)=y$ und $f(x)=y'$.\\
\textbf{Zu zeigen:} $y=y'$.\\
Aus $f(x)=y$ und $f(x)=y'$
folgt $(x,y),(x,y')\in R$ per Konstruktion von $f$.\\
Eigenschaft \punktlabel{2} besagt, dass $y=y'$.
\end{kompaktenum}
Darum ist $f$ eine Abbildung zwischen $M$ und $N$
und $\graph(f)=R$.
\end{proof}
\end{einzug}
%% SKA 4-3
\let\altsectionname\sectionname
\def\sectionname{SKA}
\section[Aufgabe 3]{}
\label{ska:4:ex:3}
\let\sectionname\altsectionname
Sei $X=\{a,b,c\}$ und betrachte die binäre Relation,
$(\Pot(X),\leq)$, definiert durch
\begin{mathe}[mc]{rcl}
A\leq B &\Longleftrightarrow &X\ohne A\subseteq X\ohne B\\
\end{mathe}
für $A,B\in\Pot(X)$.
\begin{claim*}
$(\Pot(X),\leq)$ ist eine partielle Ordnung (auch »Halbordnung« genannt).
\end{claim*}
Es gibt nun 3 Ansätze, um dies zu zeigen.
\begin{proof}[Ansatz I][Ansatz I]
Beobachte, dass für $A,B\in\Pot(X)$
\begin{mathe}[mc]{rcl}
A\leq B
&\textoverset{Defn}{\Longleftrightarrow}
&X\ohne A\subseteq X\ohne B\\
&\Longrightarrow
&X\ohne (X\ohne A)\supseteq X\ohne (X\ohne B)\\
&\Longrightarrow
&A\supseteq B,
\,\text{da $A,B\subseteq X$}\\
&\Longrightarrow
&X\ohne A\subseteq X\ohne B\\
&\textoverset{Defn}{\Longleftrightarrow}
&A\leq B,\\
\end{mathe}
also $A\leq B\Leftrightarrow A\supseteq B$.
Darum kann $(\Pot(X),\leq)$ mit $(\Pot(X),\supseteq)$
identifiziert werden.
Letzteres ist bekanntermaßen eine Halbordnung.
\end{proof}
\begin{proof}[Ansatz II][Ansatz II]
Im konkreten Falle von $X=\{a,b,c\}$ können wir die Relation
durch ein \emph{Hasse-Diagramm} skizzieren:
\hraum
{\footnotesize
\begin{tikzpicture}[node distance=1cm, thick]
\pgfmathsetmacro\habst{1.5}
\pgfmathsetmacro\vabst{1}
\node (Set1) at (0*\habst,0*\vabst) {$X$};
\node (SetAB) at (-1*\habst,1*\vabst) {$\{a,b\}$};
\node (SetAC) at (0*\habst,1*\vabst) {$\{a,c\}$};
\node (SetBC) at (1*\habst,1*\vabst) {$\{b,c\}$};
\node (SetA) at (-1*\habst,2*\vabst) {$\{a\}$};
\node (SetB) at (0*\habst,2*\vabst) {$\{b\}$};
\node (SetC) at (1*\habst,2*\vabst) {$\{c\}$};
\node (Set0) at (0*\habst,3*\vabst) {$\leer$};
\draw (Set1) edge [->] (SetAB);
\draw (Set1) edge [->] (SetAC);
\draw (Set1) edge [->] (SetBC);
\draw (SetAB) edge [->] (SetA);
\draw (SetAB) edge [->] (SetB);
\draw (SetAC) edge [->] (SetA);
\draw (SetAC) edge [->] (SetC);
\draw (SetBC) edge [->] (SetB);
\draw (SetBC) edge [->] (SetC);
\draw (SetA) edge [->] (Set0);
\draw (SetB) edge [->] (Set0);
\draw (SetC) edge [->] (Set0);
\end{tikzpicture}}
\hraum
Man sieht, dass dies einen \emph{Verband} und damit insbesondere eine Halbordnung bildet.
\end{proof}
\begin{proof}[Ansatz III][Ansatz III]
Wir gehen die Axiome einer Halbordnung durch:
\begin{kompaktenum}[\rtab][\rtab]
\item[\uwave{{\itshape Reflexivität:}}]
Sei $A\in\Pot(X)$ beliebig.
\textbf{Zu zeigen:} $A\leq A$.\\
Offensichtlich gilt $X\ohne A\subseteq X\ohne A$.\\
Per Konstruktion gilt also $A\leq A$.
\item[\uwave{{\itshape Antisymmetrie:}}]
Seien $ A, A'\in\Pot(X)$ beliebig.\\
\textbf{Zu zeigen:} $A\leq A'$ und $A'\leq A$ $\Rightarrow$ $A=A'$.\\
Es gilt
\begin{mathe}[mc]{rclql}
A\leq A'\,\text{und}\, A'\leq A
&\Longleftrightarrow
&X\ohne A\subseteq X\ohne A'
\,\text{und}\,
X\ohne A'\subseteq X\ohne A
&\text{(per Konstruktion)}\\
&\Longrightarrow
&X\ohne A=X\ohne A'
&\text{(per Definition von Mengengleichheit)}\\
&\Longrightarrow
&A=A',
&\text{da $A,A'\subseteq X$}.\\
\end{mathe}
\item[\uwave{{\itshape Transitivität:}}]
Seien $A, A',(a'',b'')\in\Pot(X)$ beliebig.\\
\textbf{Zu zeigen:} $A\leq A'$ und $A'\leq A''$ $\Rightarrow$ $A\leq A''$.\\
Es gilt
\begin{mathe}[mc]{rcl}
A\leq A'\,\text{und}\, A'\leq A''
&\Longleftrightarrow
&X\ohne A\subseteq X\ohne A'
\,\text{und}\,
X\ohne A'\subseteq X\ohne A''
\,\text{(per Konstruktion)}\\
&\Longrightarrow
&X\ohne A\subseteq X\ohne A''\\
&\Longleftrightarrow
&A\leq A''
\,\text{(per Konstruktion)}.\\
\end{mathe}
\end{kompaktenum}
Darum erfüllt $(\Pot(X),\leq)$ die Axiome einer Halbordnung.
\end{proof}
%% SKA 4-4
\let\altsectionname\sectionname
\def\sectionname{SKA}
\section[Aufgabe 4]{}
\label{ska:4:ex:4}
\let\sectionname\altsectionname
Betrachten wir die Halbordnung aus \cite[Beispiel 2.4.2(2)]{sinn2020}.
Es sei also $C=\{a,b,c\}$ und
die durch folgendes \emph{Hasse-Diagramm} dargestellte Ordnungsrelation auf $\Pot(C)$:
\hraum
{\footnotesize
\begin{tikzpicture}[node distance=1cm, thick]
\pgfmathsetmacro\habst{1.5}
\pgfmathsetmacro\vabst{1}
\node (Set1) at (0*\habst,3*\vabst) {$C$};
\node (SetAB) at (-1*\habst,2*\vabst) {$\{a,b\}$};
\node (SetAC) at (0*\habst,2*\vabst) {$\{a,c\}$};
\node (SetBC) at (1*\habst,2*\vabst) {$\{b,c\}$};
\node (SetA) at (-1*\habst,1*\vabst) {$\{a\}$};
\node (SetB) at (0*\habst,1*\vabst) {$\{b\}$};
\node (SetC) at (1*\habst,1*\vabst) {$\{c\}$};
\node (Set0) at (0*\habst,0*\vabst) {$\leer$};
\draw (Set0) edge [->] (SetA);
\draw (Set0) edge [->] (SetB);
\draw (Set0) edge [->] (SetC);
\draw (SetA) edge [->] (SetAB);
\draw (SetA) edge [->] (SetAC);
\draw (SetB) edge [->] (SetAB);
\draw (SetB) edge [->] (SetBC);
\draw (SetC) edge [->] (SetAC);
\draw (SetC) edge [->] (SetBC);
\draw (SetAB) edge [->] (Set1);
\draw (SetAC) edge [->] (Set1);
\draw (SetBC) edge [->] (Set1);
\end{tikzpicture}}
\hraum
Wenn wir das Element $\leer$ von $\Pot(C)$ entfernen sieht die Struktur folgendermaßen aus
\hraum
{\footnotesize
\begin{tikzpicture}[node distance=1cm, thick]
\pgfmathsetmacro\habst{1.5}
\pgfmathsetmacro\vabst{1}
\node (Set1) at (0*\habst,3*\vabst) {$C$};
\node (SetAB) at (-1*\habst,2*\vabst) {$\{a,b\}$};
\node (SetAC) at (0*\habst,2*\vabst) {$\{a,c\}$};
\node (SetBC) at (1*\habst,2*\vabst) {$\{b,c\}$};
\node (SetA) at (-1*\habst,1*\vabst) {$\{a\}$};
\node (SetB) at (0*\habst,1*\vabst) {$\{b\}$};
\node (SetC) at (1*\habst,1*\vabst) {$\{c\}$};
\draw (SetA) edge [->] (SetAB);
\draw (SetA) edge [->] (SetAC);
\draw (SetB) edge [->] (SetAB);
\draw (SetB) edge [->] (SetBC);
\draw (SetC) edge [->] (SetAC);
\draw (SetC) edge [->] (SetBC);
\draw (SetAB) edge [->] (Set1);
\draw (SetAC) edge [->] (Set1);
\draw (SetBC) edge [->] (Set1);
\end{tikzpicture}}
\hraum
Offensichtlich hat $(\Pot(C)\ohne\{\leer\},\subseteq)$ kein kleinstes Element.
Die Menge der minimalen Elementen ist durch $\{\{a\},\{b\},\{c\}\}$ gegeben.
Also gibt es $3$ minimale Elemente.
%% SKA 4-5
\let\altsectionname\sectionname
\def\sectionname{SKA}
\section[Aufgabe 5]{}
\label{ska:4:ex:5}
\let\sectionname\altsectionname
Sei $W$ die Menge aller Wörter und $\Sigma$ die Menge aller Buchstaben.
O.\,E. können wir annehmen, dass jedes Wort $w\in W$ der Länge $|w|\geq 2$ ist.
(In Sprachen wie Englisch, Russisch, usw. ist dies nicht der Fall,
aber wir könnten diese trivialen Wörter einfach ausschließen.)
Betrachten wir die Relation $(W,\sim)$ gegeben durch
\begin{mathe}[mc]{rcl}
\eqtag[eq:1:ska:4:ex:5]
w\sim w' &:\Longleftrightarrow &f(w)=f(w),
\end{mathe}
wobei $f:W\to\Sigma$ die Abbildung mit
$f(w)=\text{1. Buchstabe in $w$}$
für alle $w\in W$ ist.
Dann per Konstruktion \uline{reduziert} $f$
die Relation $(W,\sim)$ auf $(\Sigma,=)$.
Aufgrund dessen und da $(\Sigma,=)$ eine Äquivalenzrelation ist,
ist $(W,\sim)$ automatisch eine Äquivalenzrelation auch.
Eigentlich spielt est keine Rolle, wie die Funktion, $f$, aussieht.
Solange die Reduktion \eqcref{eq:1:ska:4:ex:5} gilt,
bleibt $(W,\sim)$ eine Äquivalenzrelation.
Dies gilt also insbesondere ebenfalls,
wenn $f$ den zweitletzten Buchstaben von Wörtern berechnet.
%% SKA 4-6
\let\altsectionname\sectionname
\def\sectionname{SKA}
\section[Aufgabe 6]{}
\label{ska:4:ex:6}
\let\sectionname\altsectionname
\begin{enumerate}{\bfseries (a)}
%% SKA 4-6a
\item
\begin{mathe}[tc]{rcl}
\sum_{i=2}^{6}(-1)^{i}i^{2}
&= &(-1)^{2}\cdot 2^{2}
+(-1)^{3}\cdot 3^{2}
+(-1)^{4}\cdot 4^{2}
+(-1)^{5}\cdot 5^{2}
+(-1)^{6}\cdot 6^{2}\\
&= &4-9+16-25+36
= 22\\
\end{mathe}
%% SKA 4-6b
\item
\begin{mathe}[tc]{rcl}
\prod_{j=1}^{4}(2j-1)
&= &(2\cdot 1 - 1)
+(2\cdot 2 - 1)
+(2\cdot 3 - 1)
+(2\cdot 4 - 1)\\
&= &1-3+5-7
= -4\\
\end{mathe}
\end{enumerate}
%% SKA 4-7
\let\altsectionname\sectionname
\def\sectionname{SKA}
\section[Aufgabe 7]{}
\label{ska:4:ex:7}
\let\sectionname\altsectionname
\begin{claim}
\makelabel{claim:main:ska:4:ex:7}
Bezeichne mit $\Phi(n)$ die Aussage
\begin{mathe}[mc]{rcl}
\eqtag[eq:1:\beweislabel]
\sum_{i=1}^{n}(-1)^{i}i^{2} &= &(-1)^{n}\frac{1}{2}n(n+1).\\
\end{mathe}
Dann gilt $\forall{n\in\ntrlpos:~}\Phi(n)$.
\end{claim}
\begin{einzug}[\rtab][\rtab]
\begin{proof}
Wir zeigen \Cref{\beweislabel} stumpf per Induktion.
\begin{kompaktenum}[\rtab][\rtab]
\item[\uwave{{\bfseries Induktionsanfang:}}]
Sei $n=1$. Dann
\begin{mathe}[mc]{rcl}
\sum_{i=1}^{n}(-1)^{i}i^{2}
&= &(-1)^{1}1^{2} = -1\\
(-1)^{n}\frac{1}{2}n(n+1)
&= &(-1)^{1}\frac{1}{2}\cdot 1\cdot (1+1) = -1\\
\end{mathe}
Also gilt \eqcref{eq:1:\beweislabel}.
Also gilt $\Phi(1)$
\item[\uwave{{\bfseries Induktionsvoraussetzung:}}]
Sei $n>1$.
Angenommen, $\Phi(n-1)$ gilt.
\item[\uwave{{\bfseries Induktionsschritt:}}]
\textbf{Zu zeigen:} $\Phi(n)$ gilt, d.\,h.
Gleichung \eqcref{eq:1:\beweislabel} gilt.\\
Es gilt
\begin{mathe}[mc]{rcl}
\sum_{i=1}^{n}(-1)^{i}i^{2}
&= &\sum_{i=1}^{n-1}(-1)^{i}i^{2} + (-1)^{n}n^{2}\\
&= &(-1)^{n-1}\frac{1}{2}(n-1)(n-1+1) + (-1)^{n}n^{2}\\
&&\text{wegen der IV}\\
&= &(-1)^{n}\cdot(-\frac{1}{2}n(n-1) + n^{2})\\
&= &(-1)^{n}\cdot(-\frac{1}{2}n^{2} + \frac{1}{2}n + n^{2})\\
&= &(-1)^{n}\cdot(\frac{1}{2}n^{2} + \frac{1}{2}n)\\
&= &(-1)^{n}\frac{1}{2}n(n+1).\\
\end{mathe}
Also gilt \eqcref{eq:1:\beweislabel}.
Also gilt $\Phi(n)$.
\end{kompaktenum}
Also gilt $\forall{n\in\ntrlpos:~}\Phi(n)$.
\end{proof}
\end{einzug}
Für die Summe $\sum_{i=3}^{n}(-1)^{i}i^{2}$
ist der Ausdruck lediglich
\begin{mathe}[mc]{rcl}
\sum_{i=3}^{n}(-1)^{i}i^{2}
&= &\sum_{i=1}^{n}(-1)^{i}i^{2}-(-1)^{1}\cdot 1-(-1)^{2}2^{2}\\
&= &(-1)^{n}\frac{1}{2}n(n+1)-3\\
\end{mathe}
für alle $n\geq 3$.
Sollten wir dies per Induktion beweisen wollen,
brauchen wir lediglich im o.\,s. Beweis
den \textbf{Induktionsanfang} auf $n=3$ zu ändern.
Der Rest bleibt erhalten.
\begin{rem}
Man merkt, dass Induktion mit Deduzieren (»Ableiten«) nichts zu tun hat.
Induktion ist schließlich nur ein Werkzeug,
um Behauptungen zu \emph{verifizieren}.
Sie verschafft uns aber keine Mittel,
um \emph{auf die Behauptungen zu kommen}.
In diesem konkreten Falle wurde Vorarbeit geleistet
und \emph{direkt} argumentiert,
um auf den Ausdruck in \eqcref{eq:1:\beweislabel} zu kommen.
Ohne diese Arbeit wären wir auf diesen Ausdruck gar nicht gekommen.
In dieser Vorarbeit steckt also die eigentliche mathematische Arbeit
und dies bedarf etwas Kreativität, Intuition, usw.
Häufig reicht diese Vorarbeit aber nur,
um auf eine sinnvolle Behauptung zu kommen,
und zum Schluss runden wir dies mit Induktion ab,
um formal die behauptete Aussage zu bestätigen.
Das ist die eigentliche Rolle von Induktion als Beweismittel.
\end{rem}
%% SKA 4-8
\let\altsectionname\sectionname
\def\sectionname{SKA}
\section[Aufgabe 8]{}
\label{ska:4:ex:8}
\let\sectionname\altsectionname
\uwave{{\bfseries Kurzes Argument:}}\\
Wenn jede Farbe jeweils auf maximal $1$ Karte vorkommt,
gibt es $\leq 4\cdot 1$ Karten.
Aber $5$ Karten werden gewählt.
\uwave{{\bfseries Ausführliches Argument:}}\\
Seien
${X:=\{\clubsuit, \diamondsuit, \heartsuit, \spadesuit\}}$
die Menge der Farben und
${Y:=\{1,2,3,4,5\}}$
die Indizes der Karten.
Sei ${f:X\to\Pot(Y)}$ die Funktion,
die der Wahl entspricht, d.\,h.
\begin{mathe}[mc]{rcl}
f(x) &= &\{y\in Y\mid\text{Karte $y$ hat Farbe $x$}\}\\
\end{mathe}
für alle Farben $x\in X$.
Nun, jede Karte, $y\in Y$, hat eine Farbe, sodass $y\in f(x)$ für ein $x\in X$.
Also $Y\subseteq\bigcup_{x\in X}f(x)$.
Und per Definition $f(x)\subseteq Y$ für alle $x\in X$.
Darum $\bigcup_{x\in X}f(x)\subseteq Y$.
Also
\begin{mathe}[mc]{rcl}
Y &= &\bigcup_{x\in X}f(x)\\
\end{mathe}
Andererseits sind die Mengen $(f(x))_{x\in X}$ paarweise disjunkt,
da jede Karte höchstens eine Farbe hat.
Also ist $(f(x))_{x\in X}$ eine \emph{Partition} von $Y$.
Darum
\begin{mathe}[mc]{ll}
&|Y| = |\bigcup_{x\in X}f(x)|
= \sum_{x\in X}|f(x)|
\leq |X|\cdot\max_{x\in X}|f(x)|\\
\Longrightarrow
&\max_{x\in X}|f(x)| \geq |Y|/|X| = 5/4 > 1\\
\Longrightarrow
&\exists{x\in X:~}|f(x)|>1\\
\Longrightarrow
&\exists{x\in X:~}|f(x)|\geq 2\\
\end{mathe}
Nach der Definition von $f$ heißt dies,
es gibt eine Farbe, $x\in\{\clubsuit, \diamondsuit, \heartsuit, \spadesuit\}$,
so dass $\geq 2$ der gezogenen Karten der Farbe $x$ sind.
%% SKA 4-9
\let\altsectionname\sectionname
\def\sectionname{SKA}
\section[Aufgabe 9]{}
\label{ska:4:ex:9}
\let\sectionname\altsectionname
\uwave{{\bfseries Kurzes Argument:}}\\
Wenn jeder Kalendartag jeweils von maximal $17$ Studierenden gefeiert wird,
gibt es $\leq 366\cdot 17=6222$ Studierende.
Aber es gibt $\geq 7000$ Studierende.
\uwave{{\bfseries Ausführliches Argument:}}\\
Seien
${X=\{\text{1.~Jan},\,\text{2.~Jan},\,\ldots,\,\text{31.~Dez}\}}$
die Menge der Kalendartage
und
${Y=\{x\mid x\,\text{ein/e Studierende/r an der Uni Leipzig}\}}$.
Sei ${f:X\to\Pot(Y)}$ die Funktion,
die der Wahl entspricht, d.\,h.
\begin{mathe}[mc]{rcl}
f(x) &= &\{y\in Y\mid\text{Studierende/r $y$ hat am Tag $x$ Geburtstag}\}\\
\end{mathe}
für alle Kalendartage $x\in X$.
Nun, jede/r Studierende/r, $y\in Y$, hat einen Geburtstag,
sodass $y\in f(x)$ für ein $x\in X$.
Also $Y\subseteq\bigcup_{x\in X}f(x)$.
Und per Definition $f(x)\subseteq Y$ für alle $x\in X$.
Darum $\bigcup_{x\in X}f(x)\subseteq Y$.
Also
\begin{mathe}[mc]{rcl}
Y &= &\bigcup_{x\in X}f(x)\\
\end{mathe}
Andererseits sind die Mengen $(f(x))_{x\in X}$ paarweise disjunkt,
da jede/r Studierende/r höchstens einen Geburtstag hat.
Also ist $(f(x))_{x\in X}$ eine \emph{Partition} von $Y$.
Darum
\begin{mathe}[mc]{ll}
&|Y| = |\bigcup_{x\in X}f(x)|
= \sum_{x\in X}|f(x)|
\leq |X|\cdot\max_{x\in X}|f(x)|\\
\Longrightarrow
&\max_{x\in X}|f(x)| \geq |Y|/|X| \geq 7000/366 > 19\\
\Longrightarrow
&\exists{x\in X:~}|f(x)|>19\\
\Longrightarrow
&\exists{x\in X:~}|f(x)|\geq 20\\
\end{mathe}
Nach der Definition von $f$ heißt dies,
es gibt einen Kalendartag, ${x\in\{\text{1.~Jan},\,\text{2.~Jan},\,\ldots,\,\text{31.~Dez}\}}$,
so dass mindestens $20$ Studierende $x$ als Geburtstag feiern.
Insbesondere gibt es $18$ Menschen, die den gleichen Geburtstag feiern.
%% SKA 4-10
\let\altsectionname\sectionname
\def\sectionname{SKA}
\section[Aufgabe 10]{}
\label{ska:4:ex:10}
\let\sectionname\altsectionname
\begin{claim}
\makelabel{claim:main:ska:4:ex:10}
Bezeichne mit $\Phi(n)$ die Aussage
\begin{kompaktitem}[\rtab][\rtab]
\item
Für alle endlichen Mengen, $E_{1},E_{2},\ldots,E_{n}$,
gilt $|\prod_{i=1}^{n}E_{i}|=\prod_{i=1}^{n}|E_{i}|$.
\end{kompaktitem}
Dann gilt $\forall{n\in\ntrlpos:~}\Phi(n)$.
\end{claim}
\begin{einzug}[\rtab][\rtab]
\begin{proof}
Wir zeigen \Cref{\beweislabel} per Induktion.
Als Induktionsanfang widmen wir uns den Fällen $n\leq 2$.
\begin{kompaktenum}[\rtab][\rtab]
\item[\uwave{{\bfseries Induktionsanfang:}}]
Sei $n=1$. Dann für alle Mengen, $E_{1}$
\begin{mathe}[mc]{rcccl}
|\prod_{i=1}^{1}E_{i}|
&= &|E_{1}|
&= &\prod_{i=1}^{1}|E_{i}|\\
\end{mathe}
Also gilt $\Phi(1)$.
\item[]
Sei $n=2$. Dann gilt für alle endlichen Mengen $E_{1},E_{2}$
\begin{mathe}[mc]{rcccccl}
|\prod_{i=1}^{2}E_{i}|
&= &|E_{1}\times E_{2}|
&= &|E_{1}|\cdot|E_{2}|
&= &\prod_{i=1}^{2}|E_{i}|.\\
\end{mathe}
(Dieses Resultat haben wir in \Cref{lemm:1:ska:4:ex:10} ausgelagert.)\\
Also gilt $\Phi(2)$.
\item[\uwave{{\bfseries Induktionsvoraussetzung:}}]
Sei $n>2$.
Angenommen, $\Phi(k)$ gilt für alle $k<n$.
\item[\uwave{{\bfseries Induktionsschritt:}}]
Seien $E_{1},E_{2},\ldots,E_{n}$ beliebige endliche Mengen.
\textbf{Zu zeigen:} $|\prod_{i=1}^{n}E_{i}|=\prod_{i=1}^{n}|E_{i}|$ gilt.\\
Es gilt
\begin{mathe}[mc]{rclql}
|\prod_{i=1}^{n}E_{i}|
&= &|\prod_{i=1}^{n-1}E_{i}\times E_{n}|\\
&= &|\prod_{i=1}^{n-1}E_{i}|\cdot|E_{n}|,
&\text{da $\Phi(2)$ gilt}\\
&= &\prod_{i=1}^{n-1}|E_{i}|\cdot|E_{n}|
&\text{wegen der IV}\\
&= &\prod_{i=1}^{n}|E_{i}|.\\
\end{mathe}
Also gilt $\Phi(n)$.
\end{kompaktenum}
Also gilt $\forall{n\in\ntrlpos:~}\Phi(n)$.
\end{proof}
\end{einzug}
Wir müssen noch den Fall für $2$ Mengen beweisen.
\begin{lemm}
\makelabel{lemm:1:ska:4:ex:10}
Seien $X$, $Y$ beliebige \uline{endliche} Mengen.
Dann $|X\times Y|=|X|\cdot |Y|$.
\end{lemm}
\begin{einzug}[\rtab][\rtab]
\begin{proof}
Wir beweisen dies per Induktion über $|Y|$ durchführen.
\begin{kompaktenum}[\rtab][\rtab]
\item[\uwave{{\bfseries Induktionsanfang:}}]
Sei $Y$ eine endliche Menge mit $|Y|=0$.
Also $Y=\leer$.
Darum
\begin{mathe}[mc]{rcccccccccl}
|X\times Y|
&= &|X\times\leer|
&= &|\leer|
&= &0
&= &|X|\cdot 0
&= &|X|\cdot|Y|.\\
\end{mathe}
\item[]
Sei $Y$ eine $1$-elementige Menge.
Dann $Y=\{y\}$ für ein Objekt, $y$.
Es ist einfach zu sehen, dass
${x\in X\mapsto (x,y)\in X\times Y}$
eine Bijektion ist.
Folglich sind $X$ und $X\times Y$ gleichmächtig.
D.\,h. $|X\times Y|=|X|=|X|\cdot 1=|X|\cdot|Y|$.
\item[\uwave{{\bfseries Induktionsvoraussetzung:}}]
Sei $n>1$.
Angenommen, $|X\times Y'|=|X|\cdot |Y'|$
für alle $k$-elementigen Mengen, $Y'$
und für alle $k<n$.
\item[\uwave{{\bfseries Induktionsschritt:}}]
Sei $Y$ eine beliebige $n$-elementige Menge.\\
\textbf{Zu zeigen:} $|X\times Y|=|X|\times|Y|$ gilt.\\
Da $n>0$, können wir ein beliebiges $y_{0}\in Y$ fixieren.\\
Setze $Y':=Y\ohne\{y_{0}\}$.
Da $Y'$ $n-1$-elementig ist, gilt per Induktionsvoraussetzung
$|X\times Y'|=|X|\cdot|Y'|=|X|\cdot(n-1)$.\\
Wegen Disjunktheit von $Y'$ und $\{y_{0}\}$,
sind $X\times Y'$ und $X\times\{y_{0}\}$ ebenfalls disjunkt.
Es folgt
\begin{longmathe}[mc]{RCLqL}
|X\times Y|
&= &|X\times (Y'\cup\{y_{0}\}|\\
&= &|(X\times Y')\cup (X\times\{y_{0}\})|\\
&= &|X\times Y'| + |X\times\{y_{0}\}|
&\text{wegen Disjunktheit}\\
&= &|X|\cdot(n-1) + |X|\cdot 1
&\text{wegen Fall für $1$-elem. Mengen}\\
&= &|X|\cdot n
&\text{wegen rekursiver Defn von Multiplikation}\\
&= &|X|\cdot |Y|.\\
\end{longmathe}
\end{kompaktenum}
Darum gilt $|X\times Y|=|X|\cdot|Y|$ für alle Mengen $X,Y$.
\end{proof}
\end{einzug}
%% SKA 4-11
\let\altsectionname\sectionname
\def\sectionname{SKA}
\section[Aufgabe 11]{}
\label{ska:4:ex:11}
\let\sectionname\altsectionname
Um ein Argument zurückzuweisen, reicht es häufig aus,
das Argument einfach \emph{ausführlich} aufzuschreiben.
Wir nehmen die Ausführung und formalisieren diese:
\begin{claim*}
Bezeichne mit $G(x)$, dass $x$ ein Goldfisch ist.
Für $n\in\ntrlpos$ bezeichne mit $\Phi(n)$ folgende Aussage
\begin{kompaktitem}[\rtab][\rtab]
\item
Für alle $n$-elementigen Mengen, $X$, von Fischen,
wenn $\exists{x\in X:~}G(x)$,
dann $\forall{x\in X:~}G(x)$.
\end{kompaktitem}
Dann $\forall{n\in\ntrlpos:~}\Phi(n)$
\end{claim*}
\begin{proof}[ungültiges Argument]
Dies wird per Induktion argumentiert.
\begin{kompaktenum}[\rtab][\rtab]
\item[\uwave{{\bfseries Induktionsanfang:}}]
Betrachte eine $1$-elementige Menge, $X$, von Fischen.\\
Angenommen, ein $x_{0}\in X$ mit $G(x_{0})$ existiere.\\
Da $X$ nur dieses eine Element enthält,
gilt offensichtlich $\forall{x\in X:~}G(x)$.
\item[\uwave{{\bfseries Induktionsvoraussetzung:}}]
Sei $n\in\ntrlpos$ mit $n>1$.
Angenommen, $\Phi(k)$ gilt für alle $k<n$.
\item[\uwave{{\bfseries Induktionsschritt:}}]
Sei $X$ eine $n$-elementige Menge von Fischen.\\
Angenommen, ein $x_{0}\in X$ mit $G(x_{0})$ existiere.
\textbf{Zu zeigen:} Für alle $x\in X$ gilt $G(x)$.\\
Setze $X_{1}:=X\ohne\{x_{0}\}$, was nicht leer ist, weil $n\geq 2$.\\
Fixiere einen Fisch $x_{1}\in X_{1}$
und setze $X_{0}:=X\ohne\{x_{1}\}$.\\
Da $x_{1}\neq x_{0}$ sind $X_{0},X_{1}$ verschiedene $n-1$-elementige Mengen:
\hraum
{\footnotesize
\begin{tikzpicture}[node distance=1cm, thick]
\pgfmathsetmacro\habst{1.5}
\pgfmathsetmacro\vabst{1.5}
\pgfmathsetmacro\rad{1.5}
\node (PtBL) at (-1.25*\habst,0*\vabst) {};
\node (PtTL) at (-1.25*\habst,2*\vabst) {};
\node (PtBR) at (+1.25*\habst,0*\vabst) {};
\node (PtTR) at (+1.25*\habst,2*\vabst) {};
\node (X0mid) at (-0.25*\habst,1*\vabst) {};
\node (X1mid) at (+0.25*\habst,1*\vabst) {};
\node[label=above:{$x_{0}$}] (x0) at (-1*\habst,1*\vabst) {$\bullet$};
\node[label=above:{$x_{1}$}] (x1) at (+1*\habst,1*\vabst) {$\bullet$};
\node[above right = 0.3*\rad and 0.3*\rad of X0mid,label=below:{$\tilde{x}$}] {$\bullet$};
\node[above left = 0.7*\rad and 0.7*\rad of X0mid] {$X_{0}$};
\node[above right = 0.7*\rad and 0.7*\rad of X1mid] {$X_{1}$};
\draw [thick, decoration={brace, mirror, raise=1*\vabst}, decorate] node [pos=0.5, anchor=north, yshift=-10pt] {$X$} (PtBL.south) -- (PtBR.south);
\draw[pattern=north west lines] (X0mid) circle[radius=1*\rad];
\draw (X1mid) circle[radius=1*\rad];
\end{tikzpicture}}
\hraum
Fokussieren wir uns zunächst auf $X_{0}$ (die schattierte Teilmenge).\\
Da $X_{0}$ $n-1$-elementig ist und $x_{0}\in X_{0}$ und $G(x_{0})$,
gilt per IV (\textdagger)~$\forall{x\in X_{0}:~}G(x)$.
\fbox{Wähle nun einen Fisch, $\tilde{x}\in X_{0}$, mit $\tilde{x}\neq x_{0}$}
und setze $X':=X\ohne\{\tilde{x}\}$ .\\
Dann ist $X'$ eine $n-1$-elementige Menge und $x_{0}\in X_{1}$ und $G(x_{0})$.\\
Per IV gilt also $\forall{x\in X':~}G(x)$.\\
Daraus und aus (\textdagger) folgt $\forall{x\in X:~}G(x)$, da ja $X=X_{0}\cup X'$.\footnote{
Per Wahl gilt $\tilde{x}\in X_{0}=X\ohne x_{1}$.
Also, $\tilde{x}\neq x_{1}$.
Also, $x_{1}\in X'$.
Also, $X=X_{0}\cup\{x_{1}\}\subseteq X_{0}\cup X'\subseteq X$.
}
Darum gilt $\Phi(n)$.
\end{kompaktenum}
Darum gilt $\forall{n\in\ntrlpos:~}\Phi(n)$.
\end{proof}
Das Problem mit diesem Argument steckt im Induktionsschritt an genau dieser Stelle:
\begin{quote}
\itshape
Wähle nun einen Fisch, $\tilde{x}\in X_{0}$, mit $\tilde{x}\neq x_{0}$ \ldots
\end{quote}
Im ursprünglichen Text ist dies die problematische Stelle:
\begin{quote}
\itshape
Jetzt können wir aber \uline{auch einen der Goldfische rausnehmen} und haben wieder \ldots
\end{quote}
Zurück aber zu unserer Formalisierung:\\
Diese Stelle im Argument ist nur möglich, wenn $X_{0}\ohne\{x_{0}\}$ nicht leer ist,
oder äquivalent, wenn $X_{0}\cap X_{1}$ nicht leer ist.
Das \uline{Diagramm} mag dies andeuten, aber das Diagramm täuscht.
Denn formal betrachtet muss das Element, $\tilde{x}\in X_{0}\cap X_{1}$,
verschieden von $x_{0}$ und $x_{1}$ sein.
Das setzt voraus, dass $n=|X|\geq 3$.
Aber im Induktionsschritt wurde nur $n>1$ vorausgesetzt!
Das heißt das Induktionsargument ist faul,
weil der Schritt $1\rightsquigarrow 2$ implizit übersprungen wird.
\setcounternach{part}{3}
\part{Quizzes}
\def\chaptername{Quiz}
%% ********************************************************************************
%% FILE: body/quizzes/quiz1.tex
%% ********************************************************************************
\setcounternach{chapter}{1}
\chapter[Woche 1]{Woche 1}
\label{quiz:1}
\begin{claim*}
Das LGS
\begin{mathe}[mc]{rcrcr}
-x &+ &a\cdot y &= &3\\
a\cdot x &- &4y &= &0\\
\end{mathe}
ist genau dann lösbar, wenn $a\in\reell\ohne\{\pm 2\}$.
\end{claim*}
\begin{proof}
Sei $a\in\reell$ beliebig. Wir führen das Gaußverfahren aus:
\begin{algorithm}[\rtab][\rtab]
Ursprüngliches LGS $(A_{\alpha}|b_{\beta})$:
\begin{mathe}[mc]{c}
\begin{matrix}{cc|c}
-1 &a &3\\
a &-4 &0\\
\end{matrix}\\
\end{mathe}
Wende die Zeilentransformationen
${Z_{2}\leftsquigarrow a\cdot Z_{1}+Z_{2}}$
an:
\begin{mathe}[mc]{c}
\begin{matrix}{cc|c}
1 &a &3\\
0 &a^{2}-4 &3a\\
\end{matrix}\\
\end{mathe}
\end{algorithm}
Wenn $a\in\{\pm 2\}$, ist das LGS unlösbar, da in der 2. Zeile links nur $0$ Einträge stehen und rechts $\pm 6$.\\
Wenn $a\notin\{\pm 2\}$, gibt es zwei Stufen und damit ist das LGS lösbar.\\
Also gilt die Behauptung.
\end{proof}
%% ********************************************************************************
%% FILE: body/quizzes/quiz2.tex
%% ********************************************************************************
\setcounternach{chapter}{2}
\chapter[Woche 2]{Woche 2}
\label{quiz:2}
Sei $L$ die Gerade $\{\mathbf{v}+t\mathbf{w}\mid t\in\reell\}\subseteq\reell^{3}$,
wobei
\begin{mathe}[mc]{rclqrcl}
\mathbf{v} &= &\begin{svector}-4\\2\\5\\\end{svector},
&\mathbf{w} &= &\begin{svector}2\\-6\\12\\\end{svector}.\\
\end{mathe}
\begin{enumerate}{\bfseries (1)}
%% QUIZ 2-a
\item
\begin{claim*}
Der Punkt, $\mathbf{x}=\begin{svector}-3\\-1\\11\\\end{svector}$, liegt in der Geraden, $L$.
\end{claim*}
\begin{proof}
Es gilt
\begin{mathe}[mc]{rcl}
\mathbf{x}\in L
&\Longleftrightarrow
&\exists{t\in\reell:~}
\mathbf{x}=\mathbf{v}+t\mathbf{w}\\
&\Longleftrightarrow
&\exists{t\in\reell:~}
\mathbf{x}-\mathbf{v}=t\mathbf{w}\\
&\Longleftrightarrow
&\exists{t\in\reell:~}
\begin{svector}1\\-3\\6\\\end{svector}=t\begin{svector}2\\-6\\12\\\end{svector}\\
\end{mathe}
Nun ist die letzte Aussage wahr,
da der Ausdruck innerhalb des Existenzquantors offensichtlich unter $t=\frac{1}{2}$ wahr ist.
Darum gilt $\mathbf{x}\in L$.
\end{proof}
%% QUIZ 2-b
\item
Fixiere einen Vektor, $\mathbf{w}_{\perp}\in\reell^{3}$,
der zu $\mathbf{w}$ normal ist.
Z.\,B. können wir
\begin{mathe}[mc]{rcl}
\mathbf{w}_{\perp} &= &\begin{svector}3\\-1\\0\\\end{svector}\\
\end{mathe}
wählen. Dann gilt $\brkt{\mathbf{w},\mathbf{w}_{\perp}}=0$,
sodass die Vektoren normal zueinander stehen.
Nun, für $\mathbf{x}\in L$ setze
\begin{mathe}[mc]{rcl}
L_{\mathbf{x}} &:= &\{\mathbf{x}+s\cdot\mathbf{w}_{\perp}\mid s\in\reell\}.\\
\end{mathe}
Dann gilt offensichtlich $\mathbf{x}\in L\cap L_{\mathbf{x}}$.\\
Andererseits, da die Richtungsvektoren in den Geraden nicht linear abhängig sind,
(da sie normal zueinander stehen),
gilt $|L\cap L_{\mathbf{x}}|\leq 1$.\\
Darum gilt $L\cap L_{\mathbf{x}}=\{\mathbf{x}\}$.
\end{enumerate}
%% ********************************************************************************
%% FILE: body/quizzes/quiz3.tex
%% ********************************************************************************
\setcounternach{chapter}{3}
\chapter[Woche 3]{Woche 3}
\label{quiz:3}
\begin{enumerate}{\bfseries (a)}
%% QUIZ 3-a
\item
\begin{claim*}
Seien $X$, $Y$ beliebige Mengen und $f:X\to Y$ eine Funktion.
Sei $B\subseteq Y$ beliebig.
Dann gilt $f(f^{-1}(B))=f(X)\cap B$.
Insbesondere gilt $f(f^{-1}(B))\subseteq B$
\end{claim*}
\begin{proof}
Für $y\in Y$ gilt
\begin{mathe}[mc]{rcl}
y\in f(f^{-1}(B))
&\Longleftrightarrow
&\exists{x\in f^{-1}(B):~}f(x)=y\\
&\Longleftrightarrow
&\exists{x\in X:~}(x\in f^{-1}(B)\,\text{und}\,f(x)=y)\\
&\Longleftrightarrow
&\exists{x\in X:~}(f(x)=y\,\text{und}\,x\in f^{-1}(B))\\
&\Longleftrightarrow
&\exists{x\in X:~}(y=f(x)\,\text{und}\,f(x)\in B)\\
&\Longleftrightarrow
&\exists{x\in X:~}(y=f(x)\,\text{und}\,y\in B)\\
&\Longleftrightarrow
&(\exists{x\in X:~}y=f(x))\,\text{und}\,y\in B\\
&\Longleftrightarrow
&y\in f(X)\,\text{und}\,y\in B\\
&\Longleftrightarrow
&y\in f(X)\cap B.\\
\end{mathe}
Darum gilt $f(f^{-1}(B))=f(X)\cap B\subseteq B$.
\end{proof}
%% QUIZ 3-b
\item
Aus (a) folgt:
\begin{kompaktitem}
\item
$f$ \uline{surjektiv} $\Longrightarrow$
$f(f^{-1}(B))=f(X)\cap B=Y\cap B=B$
für alle $B\subseteq Y$;
\item
$f$ \uline{nicht surjektiv} $\Longrightarrow$
$f(f^{-1}(Y))=f(X)\cap Y=f(X)\subset Y$ (strikt).
\end{kompaktitem}
Darum ist es notwendig und hinreichend, eine nicht-surjektive Funktion als Beispiel zu nehmen.
Hier ein minimales Beispiel $X=\{0\}$ und $Y=\{1,2\}$ und $B=Y$ und $f:X\to Y$ definiert durch $f(0)=1$.
Dann $f(f^{-1}(B))=f(f^{-1}(Y))=f(X)=\{1\}\subset Y$ (strikt).
\end{enumerate}
%% ********************************************************************************
%% FILE: back/index.tex
%% ********************************************************************************
\bibliographystyle{alpha}
\def\bibname{Literaturverzeichnis}
\nocite{*}
\bgroup
\footnotesize
%% ********************************************************************************
%% FILE: ./back/quelle.bib
%% ********************************************************************************
\begin{thebibliography}{EFT18}
\bibitem[EFT18]{ebbinghaus2018}
Heinz-Dieter Ebbinghaus, J\"org Flum, and Wolfgang Thomas.
\newblock {\em {Einf\"uhrung in die mathematische Logik}}.
\newblock 2018.
\bibitem[Jec97]{jech1997}
Thomas Jech.
\newblock {\em {Set Theory}}.
\newblock Springer-Verlag, 1997.
\bibitem[Sin20]{sinn2020}
Rainer Sinn.
\newblock {Lineare Algebra I: Skript zur Veranstaltung Universit\"at Leipzig}.
\newblock Vorlesungsskript, 2020.
\bibitem[Wal16]{waldmann2016}
Stefan Waldmann.
\newblock {\em {Lineare Algebra 1: Die Grundlagen f\"ur Studierende der
Mathematik und Physik}}.
\newblock Springer Berlin Heidelberg, 2016.
\end{thebibliography}
\egroup
\end{document}